You are on page 1of 57

1. With reference to the interior structure of the 4.

Which of the following are marine protected


earth, consider the following statements: areas in India?
1. Molten iron in the core is responsible for
1. Wildlife Sanctuary Pitti
Earth's magnetic field.
2. Wildlife Sanctuary Coringa
2. Earth's magnetic field is gradually
becoming weaker with time. 3. Wildlife Sanctuary Gahirmatha
Which of the statements given above is/are Select the correct answer using the code
correct? given below.
(a) 1 only (a) 1, 2 and 3
(b) 2 only
(b) 1 and 2 only
(c) Both 1 and 2
(c) 1 only
(d) Neither 1 nor 2
(d) 2 and 3 only
2. Consider the following statements:
1. India ranks fourth in the world in terms 5. Isostatic equilibrium is an ideal state where
of wind energy installed capacity. the crust and mantle would settle in absence
2. The state of Tamil Nadu leads in the of disturbing forces. Which of the following
production of wind energy in India.
forces can disturb the equilibrium?
3. Under Paris Agreement, India has
pledged a target of 100 GW of wind 1. Mountain building
energy generation by 2022. 2. Glaciation
Which of the statements given above is/are 3. Volcanism
correct? 4. Sedimentation
(a) 1 and 2 only
Select the correct answer using the code
(b) 2 only
given below.
(c) 2 and 3 only
(d) 1, 2 and 3 (a) 1 and 3 only
(b) 2 and 3 only
3. Consider the following statements related to (c) 1 and 4 only
the locust infestation: (d) 1, 2, 3 and 4
1. They originate in Rajasthan as Deserts
provide the perfect bare ground for
6. With reference to the International Criminal
laying eggs.
Court (ICC), consider the following
2. The rainfall in these regions will
produce enough green vegetation to statements:
enable both egg-laying and hopper 1. It is the world’s first permanent
development. international criminal court.
3. In India Ministry of Earth Sciences is 2. It is a non-UN organization.
responsible for monitoring, survey, and
3. Its judgments can be appealed against.
control of Desert Locust.
Which of the statements given above is/are
Which of the statements given above are
correct? correct?
(a) 1 and 2 only (a) 1 and 2 only
(b) 2 and 3 only (b) 3 only
(c) 1 and 3 only
(c) 2 and 3 only
(d) 1, 2 and 3
(d) 1, 2 and 3

https://t.me/upsc_pt_mains
2 ©Vision IAS
7. With reference to the occurrence of minerals 11. Confederation of Indian Industry's Green
across India, consider the following pairs: Building Congress 2020 was recently
Mineral Mine inaugurated. In this context, consider the
1. Copper : Khetri following statements regarding green
2. Aluminum : Balaghat buildings rating systems in India:
3. Iron Ore : Gurumahisani 1. Green Rating for Integrated Habitat
4. Manganese : Panchpatmali Hills Assessment (GRIHA) is the only rating
Which of the pairs given above are correctly tool for green buildings in India.
matched? 2. The Indian Green Building Council
(a) 1, 2 and 3 only develops new green building rating
(b) 2 and 4 only programmes.
(c) 1 and 3 only Which of the statements given above is/are
(d) 1, 2, 3 and 4 correct?
(a) 1 only
8. These are immature soils which overlie the (b) 2 only
Vindhayan sandstones and Ladakh shales. (c) Both 1 and 2
Their color ranges from light yellow to deep (d) Neither 1 nor 2
brown with a thickness of 7 to 15 cm. These
soils contain 35 per cent or more, by volume, 12. If you are travelling in India from North to
of rock fragments, cobbles, gravel, and South, in which of the following orders are
laterite concretions or ironstones within you likely to meet the following tribes?
shallow depths. 1. Chenchu
Which of the following soils is best 2. Shompen
described by the above passage? 3. Sahariya
(a) Peaty Soils 4. Pangwala
(b) Skeletal Soils Select the correct answer using the code
(c) Alluvial Soils given below.
(a) 2-1-3-4
(d) Black Soils
(b) 2-3-1-4
(c) 4-3-1-2
9. With reference to India's biodiversity,
(d) 3-1-4-2
Eriocaulon parvicephalum, Eriocaulon
karaavalense, Ardisia ramaswamii recently
13. Arrange the following commodities in
seen in the news are species of
increasing order of their share in India's
(a) Birds
export basket:
(b) Reptiles
1. Petroleum Products
(c) Plants
2. Precious and Semi-precious stones
(d) Amphibians
3. Iron and Steel
4. Drug formulations
10. In the context of environmental protection,
Select the correct answer using the code
the Waigani Convention deals with
given below.
(a) Ocean acidification
(a) 3-4-2-1
(b) Ozone-depleting substances
(b) 2-3-1-4
(c) Wildlife Protection
(c) 3-2-4-1
(d) Hazardous Wastes
(d) 1-2-4-3

3 www.visionias.in ©Vision IAS


14. Which of the following rules deal with the 17. Which of the following is/are incorporated
phasing-out of chlorinated plastics bags? under the ambit of Over-the-top media
1. Hazardous Waste Management Rules, services (OTT) platforms?
1. Audio streaming
2016
2. Messaging services
2. Bio-Medical Waste Management
3. Internet-based voice calling solutions
(Amendment) Rules, 2018 Select the correct answer using the code
3. Plastic waste management rules 2016 below.
Select the correct answer using the code (a) 1 and 3 only
given below. (b) 2 only
(a) 1 only (c) 2 and 3 only
(d) 1, 2 and 3
(b) 2 only
(c) 2 and 3 only
18. ADMM-Plus sometimes seen in the news is
(d) 1, 2 and 3
a platform to strengthen security and defense
cooperation for peace, stability, and
15. Arrange the following rock system in the development of?
correct chronological order. (a) Eurasia
1. Gondwana (b) ASEAN region
2. Dharwar (c) African Union
(d) Middle East region
3. Cuddapah
4. Dravidian
19. Allelopathy refers to the process of release
Select the correct answer using the code
of chemicals from plants that
given below. (a) inhibits growth of other plants.
(a) 1 - 2 - 3 - 4 (b) results in efficient nitrogen fixation.
(b) 2 - 3 - 4 - 1 (c) is essential for storage of glucose.
(c) 4 - 3 - 1 - 2 (d) occurs as a result of transpiration.
(d) 2 - 4 - 3 - 1
20. Consider the following statements regarding
India's national inland waterways:
16. With respect to the Bonn Challenge,
1. The National Waterway 4 connecting
consider the following statements:
Kakinada to Puducherry is the shortest
1. It was launched by the World Wide
Inland water of India.
Fund for Nature. 2. The National Waterway 5 connects
2. Its goal is to bring 350 million hectares Lakhipur and Bhanga of the Barak River
of degraded and deforested landscapes in North-East India.
into restoration by 2030. 3. National Waterway 3 is the only
Which of the statements given above is/are waterway of India that serves five states.
Which of the statements given above is/are
correct?
not correct?
(a) 1 only
(a) 1 only
(b) 2 only (b) 2 and 3 only
(c) Both 1 and 2 (c) 1 and 3 only
(d) Neither 1 nor 2 (d) 1, 2 and 3
4 www.visionias.in ©Vision IAS
21. Which of the following correctly explains 24. With reference to the common occurrence of
the “Biorock technology”? various drainage patterns across different
regions, consider the following pairs:
(a) It refers to a process of recycling organic
Drainage Pattern Region
waste. 1. Rectangular : Chotanagpur
(b) It is an innovative process originally Drainage Plateau
invented to produce natural building 2. Radial Pattern : Amarkantak
Mountain
materials in the sea.
3. Dendritic : Indo-Gangetic
(c) It is a special technology used for oil- Drainage Plains
drilling. 4. Trellis Drainage : Vindhyan
Mountains
(d) None of these
Which of the pairs given above is/are
correctly matched?
22. Which of the following rivers in India pass (a) 1, 2 and 3 only
through the cities where the Kumbh Mela is (b) 2 and 3 only
observed? (c) 2 only
(d) 1, 3 and 4 only
1. Ganga
2. Godavari 25. With reference to the 'TEEB India Initiative',
3. Kaveri consider the following statements:
4. Shipra 1. It highlights the economic consequences
of the loss of biological diversity.
Select the correct answer using the code
2. It has been developed by India and
given below. France as a part of their Paris Climate
(a) 1 and 3 only Change commitments.
(b) 1 and 4 only Which of the statements given above is/are
correct?
(c) 1, 2 and 4 only
(a) 1 only
(d) 2, 3 and 4 only (b) 2 only
(c) Both 1 and 2
23. Which of the following factors increase a (d) Neither 1 nor 2

species' susceptibility to extinction?


26. With reference to the 'Waves Programme' of
1. Species belonging to low trophic level.
the World Bank, which of the following is
2. Species that rely on few prey species. correct?
3. Reliance on homogenous energy (a) It aims to set up tsunami warning system
in the Indian ocean.
channel.
(b) It provides fund for experimental
Select the correct answer using the code research in Tidal energy.
given below. (c) It aims to promote sustainable
(a) 1 only development through Wealth
Accounting and the Valuation of
(b) 1 and 2 only
Ecosystem Services.
(c) 2 and 3 only (d) It provides fund for enhancing research
(d) 1, 2 and 3 in quantum physics.

5 www.visionias.in ©Vision IAS


27. Which of the following factors can affect the 30. Consider the following statements regarding
rotational speed of Earth? Oilzapper and Oilivorous-S technologies:
1. Tides
1. Both are bioremediation technologies
2. Mean sea level
using biological organisms to reduce or
3. Wind patterns
Select the correct answer using the code eliminate toxic pollutants.
given below. 2. Both are in-situ technologies which do
(a) 1 only not require transferring large quantities
(b) 2 and 3 only
of contaminated waste from the site.
(c) 1 and 3 only
Which of the statements given above is/are
(d) 1, 2 and 3
correct?
28. With reference to the Ramsar Convention on (a) 1 only
Wetlands, which of the following state/states (b) 2 only
which does/do not have any Ramsar site in
(c) Both 1 and 2
India?
(d) Neither 1 nor 2
1. Bihar
2. Karnataka
3. Tripura 31. In the context of Indian geography, Gir,
4. Arunachal Pradesh Sindhi Sahiwal and Tharparkar are:
Select the correct answer using the code (a) Breeds of Indian cattle
given below.
(b) Tribes of Indian desert ecosystems
(a) 1 and 2 only
(c) Protected areas of threatened species
(b) 2 and 3 only
(c) 2 and 4 only (d) Jhum cultivation practices
(d) 4 only
32. With reference to the phenomenon of Lake
29. Consider the following statements: Effect Snow, consider the following
1. Biological Oxygen Demand
statements:
(BOD) measures the amount of oxygen
1. It is the snowfall caused by a cold
required by micro-organisms for the
decomposition of bio-degradable matter airmass moving over relatively warm
under aerobic condition. lake waters.
2. Chemical Oxygen Demand 2. It is commonly observed in the Great
(COD) measures both biodegradable and
Lakes region of North America.
non-biodegradable pollutants present in
Which of the statements given above is/are
the wastewater.
Which of the statements given above is/ are correct?
correct? (a) 1 only
(a) 1 only (b) 2 only
(b) 2 only
(c) Both 1 and 2
(c) Both 1 and 2
(d) Neither 1 nor 2
(d) Neither 1 nor 2
6 www.visionias.in ©Vision IAS
33. With reference to Sansad Adarsh Gram 37. Consider the following pairs regarding
Yojana, consider the following statements: international conventions:
1. MPs can select any gram panchayat of
their constituency to be developed as Convention Aim
Adarsh Gram. 1. Stockholm : It aims to protect human
2. The scheme will be funded from the
Convention health and the
Consolidated Fund of India.
3. Ministry of Panchayati Raj will be the environment from the
nodal ministry coordinating and effects of persistent
monitoring the scheme.
Which of the statements given above is/are organic pollutants
correct? (POPs).
(a) 1 only 2. Basel : It aims to ban the
(b) 1 and 2 only
(c) 1, 2, and 3 Convention movements of
(d) None radioactive waste
between nations.
34. The 'Neem Campaign', often seen in the
news with reference to India's relation with 3. Rotterdam : It aims to promote
the West, is related to: Convention shared responsibilities
(a) A collaboration between India and
in relation to the
western countries for greening India.
(b) An initiative to mobilize worldwide importation of
support to protect indigenous knowledge hazardous chemicals.
systems and resources of the Third
Which of the pairs given above are correctly
World from piracy by the West.
(c) Ban on import of non-neem coated urea matched?
from West, by India. (a) 1 and 2 only
(d) An initiative to propagate traditional
medicine practices in India. (b) 2 and 3 only
(c) 1 and 3 only
35. Consider the following pairs: (d) 1, 2 and 3
Indicators Definition
1. Stunting : low weight for height
2. Wasting : low height for age 38. Consider the following authorities that are
3. Under-weight : low weight for age created for the protection of wildlife in
Which of the pairs given above is/are
correctly matched? India:
(a) 1 and 2 only 1. National Tiger Conservation Authority
(b) 1 and 3 only 2. Central Zoo Authority
(c) 3 only
(d) 1, 2 and 3 3. National Board for Wild Life
4. State Board for Wild Life
36. Which of the following can be used as 5. National Biodiversity Authority
bioindicators?
Which of the above authorities have been
1. Plants
2. Animals formed under the Wildlife Protection Act,
3. Microorganisms 1972?
Select the correct answer using the code
given below. (a) 1 and 3 only
(a) 1 only (b) 1, 2 and 5 only
(b) 2 and 3 only
(c) 2, 3 and 5 only
(c) 3 only
(d) 1, 2 and 3 (d) 1, 2, 3 and 4

7 www.visionias.in ©Vision IAS


39. TX2 is a global commitment to double the 42. With reference to Pangong Tso Lake,
world’s wild tiger by 2022. In this context, consider the following statements:
consider the following statements regarding 1. It is a part of the Indus river basin area.
TX2: 2. It is a narrow deep endorheic
1. The goal has been set by the Convention (landlocked) lake situated in Ladakh
on International Trade in Endangered Himalayas.
Species of Wild Fauna and Flora 3. It does not freeze in winter due to high
(CITES). salinity.
2. As a part of this programme, Nepal Which of the statements given above is/are
became the first country to double its correct?
tiger population. (a) 1 and 2 only
Which of the statements given above is/are (b) 2 only
correct? (c) 2 and 3 only
(a) 1 only (d) 1, 2 and 3
(b) 2 only
(c) Both 1 and 2 43. Which of the following are the most likely
(d) Neither 1 nor 2 places to find the Nilgiri Tahr in its natural
habitat?
40. Consider the following statements regarding 1. Eravikulam National Park
Jet Streams: 2. Kishanpur Wildlife Sanctuary
1. Jet streams are stronger in winters for 3. Periyar National Park
both the northern and southern 4. Askot Wildlife Sanctuary
hemispheres. Select the correct answer using the code
2. Speed of jet streams gradually increases given below.
from its central core towards the outer (a) 1 and 2 only
boundary. (b) 1 and 3 only
3. Jet streams move along with the (c) 2 and 4 only
apparent latitudinal movement of the (d) 1, 2 and 3 only
sun.
Which of the statements given above is/are 44. With reference to the 'Malabar Civet',
correct? consider the following statements:
(a) 1 only 1. It is endemic to the North-eastern region
(b) 2 and 3 only of India.
(c) 1 and 3 only 2. It is listed as Critically Endangered on
(d) 1, 2 and 3 the IUCN Red list.
3. It is diurnal in behaviour.
41. Data free flow with trust (DFFT) is Which of the statements given above is/are
sometimes seen in the news is related to: correct?
(a) Osaka Declaration (a) 1 and 2 only
(b) Brazil Declaration (b) 2 only
(c) Montreal Declaration (c) 1 and 3 only
(d) Stockholm Declaration (d) 1, 2 and 3

8 www.visionias.in ©Vision IAS


59. River Valleys have served to be an abundant 62. With reference to the 'Pitcher Plant', consider
source of important minerals. In this context, the following statements:
consider the following pairs: 1. It is a carnivorous species of plant.
River Valley Famous for 2. It is protected under Schedule 1 of the
1. Damodar : Coal Wildlife Protection Act 1972
2. McArthur : Zinc
3. It is used as a cure for smallpox.
3. Birim : Gold
Which of the statements given above is/are
Which of the pairs given above is/are
correct?
correctly matched?
(a) 1 and 2 only
(a) 1 and 3 only
(b) 1 only (b) 1 and 3 only

(c) 2 and 3 only (c) 2 only


(d) 1, 2 and 3 (d) 1, 2 and 3

60. Which of the following particulate pollutants 63. With reference to Biological evolution,
is/are considered as the viable particulate arrange the following in correct
pollutants of the atmosphere? chronological order of their occurrence on
1. Smoke Earth:
2. Dust 1. Fishes
3. Mist 2. Reptiles
4. Fungi
3. Amphibians
Select the correct answer using the code
Select the correct answer using the code
given below.
given below.
(a) 1 and 2 only
(a) 1 - 3 - 2
(b) 3 and 4 only
(c) 1, 2 and 3 only (b) 2 - 1 - 3

(d) 4 only (c) 1 - 2 - 3


(d) 2 - 3 - 1
61. With reference to the recently published
World Migration Report 2020, consider the 64. Which of the following are the adverse
following statements effects of Ultraviolet B (UVB) rays?
1. Social unrest and violence have been the 1. Development of non-melanoma skin
major factors for international cancer.
migration. 2. Change in the composition of forest and
2. The United States of America is the top grassland species.
destination for migrants.
3. A decrease in reproductive capacity and
3. India is the largest country of origin of
impaired larval development.
international migrants.
Select the correct answer using the code
Which of the statements given above is/are
given below.
correct?
(a) 2 and 3 only
(a) 2 only
(b) 2 and 3 only (b) 1 only
(c) 1 and 3 only (c) 1, 2 and 3
(d) 1, 2 and 3 (d) 1 and 3 only

11 www.visionias.in ©Vision IAS


65. A climatic region is characterized by the 68. Consider the following statements:
following features: 1. This National park is situated in the
1. Warm moist summer and cool dry
Vindhyan Mountain range.
winter.
2. Very densely populated. 2. A tributary of the Yamuna River flows
3. Supports luxuriant vegetation. through this park.
Which of the following climatic regions is 3. It is a critical tiger habitat and a
described above?
biosphere reserve.
(a) Warm Temperate Western Margin
4. Tiger reintroduction project started here
Climate
(b) Savanna Climate in 2009.
(c) Warm Temperate Eastern Margin The above statements describe which of the
Climate following national parks?
(d) Tropical Monsoon Climate
(a) Bandhavgarh National park
(b) Kanha National Park
66. Consider the following pairs:
Panch Prayag River Confluence (c) Panna National Park
1. : Alaknanda and Dhauli (d) Pench National Park
Vishnuprayag Ganga
2. Nandpragay : Alaknanda and
69. Consider the following statements related to
Nandakini
the recently launched Cooling Action Plan:
3. : Alaknanda and
Karnaprayag Mandakini 1. It was launched by India with the only
4. : Alaknanda and Pinder aim to fulfil its aim under India's
Rudraprayag Intended Nationally Determined
5. Devprayag : Bhagirathi and
Contribution.
Alaknanda
Which of the pairs given above are correctly 2. One of its aims is to reduce cooling
matched? demand across sectors by 20% to 25%.
(a) 1, 3 and 4 only Which of the statements given above is/are
(b) 1, 2 and 5 only
correct?
(c) 2, 3 and 4 only
(a) 1 only
(d) 1, 2, 3, 4 and 5
(b) 2 only
67. Consider the following pairs: (c) Both 1 and 2
Hill Ranges State (d) Neither 1 nor 2
1. Palkonda Range : Andhra Pradesh
2. Kaimur Range : Maharashtra
70. With which of the following countries does
3. Maikal Range : Jharkhand
Which of the pairs given above is/are India have a balance of trade surplus?
correctly matched? (a) Saudi Arabia
(a) 1 only (b) United Arab Emirates
(b) 2 and 3 only
(c) Germany
(c) 1 and 3 only
(d) Hong Kong
(d) 1, 2 and 3
12 www.visionias.in ©Vision IAS
71. Consider the following statements regarding 74. With reference to different types of Shipping
United Nations Human Rights Council Ports, consider the following pairs:
(UNHRC):
Type of Port Port
1. It was established in 2006 by UN
General Assembly. 1. Dry Port : Kolkata
2. It does not have the authority to impose 2. Inland Port : Riyadh
legal penalty on the violating country. 3. Warm Water Port : Kushiro
3. Pakistan has not been re-elected due to Which of the pairs given above is/are
protests around the globe.
correctly matched?
Which of the statements given above are
correct? (a) 1 only
(a) 1 and 2 only (b) 2 and 3 only
(b) 2 and 3 only (c) 3 only
(c) 1 and 3 only (d) 1, 2 and 3
(d) 1, 2 and 3

75. Which of the following can be considered as


72. With reference to Surface Currents and Deep
Water currents, consider the following threats to aquatic biodiversity?
statements: 1. Building of Dams
1. Surface currents are largely driven by 2. Overfishing
winds, while deepwater currents are
3. Invasive species
driven by differences in water density.
4. Forestry practice including afforestation
2. Surface currents have a higher velocity
than deepwater currents. Select the correct answer using the code
Which of the statements given above is/are given below.
correct? (a) 1, 2 and 3 only
(a) 1 only
(b) 2 and 3 only
(b) 2 only
(c) Both 1 and 2 (c) 1 and 4 only
(d) Neither 1 nor 2 (d) 1, 2, 3 and 4

73. Consider the following statements with 76. Which of the following initiatives are
respect to volatile organic compounds: planned under the National Mission for
1. They are organic chemicals that have a
Enhanced Energy Efficiency?
high vapour pressure at ordinary room
temperature. 1. Perform Achieve and Trade
2. They include both man-made and 2. Market Transformation for Energy
naturally occurring chemical Efficiency
compounds. 3. Energy Efficiency Financing Platform
3. They are produced both by plants and
4. Framework for Energy Efficiency
animals.
4. The exhaled human breath also contains Economic Development
volatile organic compounds. Select the correct answer using the code
Which of the following statements given given below.
above is/are correct?
(a) 1 and 2 only
(a) 1 only
(b) 2 and 3 only (b) 3 and 4 only
(c) 1, 2 and 3 only (c) 1, 2, 3 and 4
(d) 1, 2, 3 and 4 (d) 1, 2 and 3 only
13 www.visionias.in ©Vision IAS
77. With reference to Natural Gas consumption 80. Consider the following statements regarding
in India, consider the following statements: a 'Bomb Cyclone'?
1. Majority of India's LNG is consumed by 1. It is a cyclone that rapidly intensifies
the City Gas Distribution network for only in tropical areas.
CNG fuel and PNG household needs. 2. It is accompanied by a fall in
2. The share of domestic gas and imported atmospheric temperature and pressure.

Regasified LNG is the ratio of about Which of the statements given above is/are
correct?
75% and 25%.
(a) 1 only
Which of the statements given above is/are
(b) 2 only
correct?
(c) Both 1 and 2
(a) 1 only
(d) Neither 1 nor 2
(b) 2 only
(c) Both 1 and 2
81. Which of the following are responsible for
(d) Neither 1 nor 2
the increase in the concentration of
formaldehyde in the environment?
78. The recently launched 'Meri Saheli' initiative
1. Pressed-wood products
aims to: 2. Natural gas
(a) rehabilitate and reintegrate trafficked 3. Tobacco
women. 4. Some cosmetics and personal products
(b) provide integrated support to women 5. Wallpaper and paints
affected by violence in public and Select the correct answer using the code
private spaces within family, community given below.
and at the workplace. (a) 1 and 2 only
(c) cater to the primary needs of shelter, (b) 2 and 3 only
food, and clothing of the women in (c) 1, 3 and 4 only
distress and who are without any social (d) 1, 2, 3, 4 and 5
and economic support.
(d) provide safety and security to female 82. Consider the following statements related to

passengers travelling by trains. biochar:


1. It is produced by pyrolysis of biomass.
2. It can increase the soil fertility of acidic
79. The 'International Satavadhanam' program
soils and also help to increase
has been launched virtually by the Vice
agricultural productivity.
President recently. With reference to the
3. It is used for carbon sequestration and
cultural history of India, the term
thus helps control climate change.
'avadhanam' is a:
Which of the statements given above are
(a) linguistic literary performance
correct?
(b) Sanskrit work authored by Sri
(a) 1, 2 and 3
Krishnadevaraya
(b) 1 and 2 only
(c) folk play/theatre of sangam age (c) 1 and 3 only
(d) religious ritual of Jains (d) 2 and 3 only

14 www.visionias.in ©Vision IAS


83. Consider the following statement regarding 86. Which one of the following protected areas
the provision that declares an animal as is well-known for the conservation of a sub-
vermin under the Wildlife Protection Act
1972: species of the Indian elk (Hangul) that
1. The final authority to declare any wild thrives in dense riverine forests in the high
animal a vermin is given to the State valleys and mountains?
Government.
2. As long as the notification is in force (a) Dachigam National Park
such wild animal shall be included in (b) Tal Chhapar Wildlife Sanctuary
Schedule V of the law.
(c) Mudumalai Wildlife Sanctuary
Which of the statements given above is/are
correct? (d) Kanha National Park
(a) 1 only
(b) 2 only 87. Consider the following statements regarding
(c) Both 1 and 2
(d) Neither 1 nor 2 India's population characteristics according
to the census 2011:
84. The deadline of a WHO-led initiative to 1. Maharashtra is the most populous state
eliminate Kala-Azar as a public health
in India followed by Uttar Pradesh.
problem from the South East Asia region has
now been extended from 2020 to 2023. In 2. Kerala has the highest sex ratio in India
this context, consider the statements among all states and UTs.
regarding Kala-Azar:
3. The percentage growth in literacy rates
1. It is caused by a protozoan Leishmania
parasite. compared to the previous decade was
2. It is transmitted due to contaminated higher in females than males.
water and food.
Which of the statements given above are
3. It is associated with high fatality.
Which of the statements given above is/are correct?
correct? (a) 1 and 2 only
(a) 1 and 2 only
(b) 2 and 3 only
(b) 2 and 3 only
(c) 1 and 3 only (c) 1 and 3 only
(d) 1, 2 and 3 (d) 1, 2 and 3

85. Consider the following statements regarding


88. Which of the following is/are the
The Climate Vulnerable Forum (CVF) :
1. It is an international partnership of applications of fly ash?
countries highly vulnerable to a warming 1. Construction of road embankments.
planet.
2. Soil conditioner
2. The Forum serves as a North-South
cooperation platform for participating 3. Cement Bricks
governments to act together to deal with Select the correct answer using the code
global climate change.
given below.
Which of the statements given above is/are
correct? (a) 1 and 2 only
(a) 1 only (b) 2 only
(b) 2 only
(c) 1 and 3 only
(c) Both 1 and 2
(d) Neither 1 nor 2 (d) 1, 2 and 3

15 www.visionias.in ©Vision IAS


89. Consider the following statements, with 92. Which of the following are observed during
respect to nano fertilizers: an El Niño event?
1. They are required in quantities of just a 1. More tropical storms and hurricanes in
few grams per acre as compared to the eastern Pacific
chemical fertilizers that are required in 2. Strengthening of the easterly trade winds
converging across the equatorial
kilograms per acre.
Pacific.
2. They supply nutrients slowly and
3. Reversal of ocean currents along the
gradually for a longer period of time.
equator.
Which of the statements given above is/are
Select the correct answer using the code
correct? given below.
(a) 1 only (a) 1 and 2 only
(b) 2 only (b) 2 and 3 only
(c) Both 1 and 2 (c) 1 and 3 only
(d) Neither 1 nor 2 (d) 1, 2 and 3

90. With reference to the pulsars, consider the 93. With reference to Seafloor spreading,
following statements: consider the following statements:
1. They are rapidly rotating neutron stars 1. Seafloor spreading is the result of
that emit radio frequency pulses. convection in the molten outer core.
2. They have magnetic fields stronger than 2. Seafloor spreading occurs only at
divergent plate boundaries.
that of the Earth.
3. Slowly spreading ridges form taller and
3. They are used to study the gravitational
narrower cliffs than the rapidly
waves.
spreading ridges.
Which of the statements given above is/are
Which of the statements given above are
correct? correct?
(a) 1 and 2 only (a) 1 and 2 only
(b) 2 and 3 only (b) 2 and 3 only
(c) 1 and 3 only (c) 1 and 3 only
(d) 1, 2 and 3 (d) 1, 2 and 3

91. Which of the following is/are the advantages 94. With reference to the World Food
of mixed subsistence farming? Programme, consider the following
1. There is larger diversity of agricultural statements:
output 1. It is an initiative of the Food and
Agriculture Organization (FAO) and
2. Farmers need not have complete
Project Concern International (PCI).
knowledge of mixed farming
2. WFP has a dedicated and independent
3. Provides farmers with a stable source of
source of funds from UN member
income
countries.
Select the correct answer using the code Which of the statements given above is/are
given below. correct?
(a) 1 and 2 only (a) 1 only
(b) 2 only (b) 2 only
(c) 1 and 3 only (c) Both 1 and 2
(d) 1, 2 and 3 (d) Neither 1 nor 2

16 www.visionias.in ©Vision IAS


95. Consider the following pairs: 97. Consider the following pairs:
Major Shipping Link Between
Wetland Type Description
Canals
1. Carr : Waterlogged wooded 1. Corinth : White Sea and Baltic
Sea
terrain
2. Kiel : Baltic Sea and North
2. Bog : Wetland that Sea
3. Suez : Mediterranean Sea
accumulates peat due
and Persian Gulf
to incomplete 4. Panama : Pacific and Atlantic
Ocean
decomposition of plant
Which of the pairs given above are correctly
matter matched?
(a) 1 and 4 only
3. Marsh : Wetland that is
(b) 1, 2 and 3 only
dominated by (c) 2 and 4 only
(d) 1, 2, 3 and 4
herbaceous rather than
woody plant species 98. “Century Plant”, “Shute Creek Gas
Which of the pairs given above are correctly Processing Plant”, and “Great Plains
Synfuels Plant” are related to which of the
matched? following?
(a) 1 and 2 only (a) Nuclear Power Generator
(b) Carbon Capture and Storage
(b) 2 and 3 only (c) Solar Power Generator
(c) 1 and 3 only (d) Coal Storage and Processing
(d) 1, 2 and 3
99. This is a sedentary pastoral tribe of
southwestern Ethiopia living on the banks of
96. Consider the following statements regarding the Omo River. They have a fierce culture,
DAP (Diammonium Phosphate), MOP with a preference for stick fighting, also
known as Donga. Piercings and lip plates are
(Muriate of Potash) and Urea based an important feature of their culture.
fertilizers in India: Which of the following tribe is being
described in the above passage?
1. There is a greater demand for DAP and (a) Masai
MOP fertilizers than Urea in India. (b) Suri
(c) Hamer
2. India is self-sufficient in the production (d) Angami
of Urea as well as MOP based fertilizers.
3. Urea consumption in India during the 100. Which of the following are the necessary
conditions for the formation of a tropical
Rabi season is twice that in the Kharif cyclone?
crop season. 1. Warm ocean waters
2. Atmospheric stability
Which of the statements given above is/are 3. Thermally contrasting air masses
correct? 4. Moisture in the mid-troposphere
Select the correct answer using the code
(a) 1 only given below.
(b) 2 and 3 only (a) 2 and 3 only
(b) 1, 2 and 3 only
(c) 1 and 3 only (c) 1 and 4 only
(d) None (d) 1, 2, 3 and 4

1 ©Vision IAS
VISIONIAS
www.visionias.in
ANSWERS & EXPLANATIONS
GENERAL STUDIES (P) TEST – 3197 (2021)

Q 1.C
• The internal structure of Earth is divided into concentric shells: an outer silicate solid crust, a highly
viscous asthenosphere and solid mantle, a liquid outer core and a solid inner core.
• The Earth's core is gradually cooling down over time, and eventually, it will solidify completely. Since the
Earth's magnetic field (which protects the atmosphere and biosphere from harmful radiation) is generated
by molten iron in the core, the solidification of the core will gradually diminish the Earth Magnetic
field. Hence, both statements are correct.
• In the last 200 years, the electromagnetic field around Earth has lost around nine percent of its strength.
Between 1970 and 2020, the magnetic field of Earth has weakened considerably in the region stretching
from Africa to South America, which is known as the 'South Atlantic Anomaly'.
• When the Earth formed, it would have been entirely molten due to the release of gravitational energy; at
this time, the Earth became chemically differentiated, meaning that heavy elements (notably iron) mostly
sank to the center to form the core while relatively light elements remained in the mantle and crust.
• The most important radioactive species on Earth -- uranium-235 and -238, thorium-232, and potassium-40
-- are lithophilic or 'rock-loving.' Lithophilic elements readily form chemical bonds with oxygen, which
helps them to remain in the crust and mantle while others sink to the core. Hence option (c) is the correct
answer.

Q 2.A
• Wind energy is basically harnessing of wind power to produce electricity. The kinetic energy of the wind
is converted to electrical energy. Wind power in the form of traditional windmills is used -for grinding
corn, pumping water, sailing ships and harnessed to generate electricity in a larger scale with better
technology.
• India has the second-highest wind capacity in Asia and fourth highest in the world and is the only
Asian country apart from China to make the list, with a total capacity of 37GW.
• The country has the third and fourth largest onshore wind farms in the world, the Muppandal wind farm in
Tamil Nadu, Southern India (1,500MW) and the Jaisalmer Wind Park in Rajasthan, Northern India
(1,064MW).
• Tamil Nadu with 9231.77 MW of installed wind capacity is well ahead of the rest and second
positioned Gujarat which has 7203.77 MW of wind generation capacity.
• The Indian government has set a target of installing 60GW of wind energy by 2022, as pledged in its
Intended nationally determined contributions (INDC). Hence the correct answer is option (a).

Q 3.A
• Indian Agriculture is highly prone to Desert Locust. The Desert Locust is a trans-boundary pest that can
cause irreparable damages.
• Locust Control and Research is a sub-component under the Sub Mission on Plant Protection and Plant
Quarantine (SMPPQ), a scheme under Green Revolution (Krishonnati Yojana) through which regulatory,
monitoring, surveillance, and capacity building functions are performed.
• In order to keep the menace of locust at bay Locust Warning Organization (LWO) has been established.
• Locust Warning Organisation (LWO), Directorate of Plant Protection Quarantine and
Storage, Ministry of Agriculture & Farmers Welfare, Department of Agriculture, Cooperation &
Farmers Welfare is responsible for monitoring, survey and control of Desert Locust in Scheduled Desert
Areas mainly in the States of Rajasthan and Gujarat. Hence statement 3 is not correct.
1 www.visionias.in ©Vision IAS
• Locusts normally live and breed in semi-arid/desert regions. For laying eggs, they require bare ground,
which is rarely found in areas with dense vegetation. So, they are more likely to breed in Rajasthan than in
the Indo-Gangetic plains or the Godavari and Cauvery delta. Hence statement 1 is correct.
• While green vegetation is good for hopper development – the stage between the nymph that has hatched
and before its turning into a winged adult moth – such cover isn’t widespread enough in deserts to allow
the growth of large locust populations. Hence statement 2 is correct.

Q 4.A
• A marine protected area (MPA) is essentially a space in the seas, oceans, estuaries where human
activities are more strictly regulated than the surrounding waters - similar to parks we have on land. These
places are given special protections for natural or historic marine resources by local, state, territorial,
native, regional, or national authorities.
• Pakshipitti (pakshi meaning "bird" in Malayalam, Telugu and Tamil), is an uninhabited coral islet
in the Union Territory of Lakshadweep, India. Pitti Bird Sanctuary is a small reef that is approximately
24 north-west of Kavaratti, is one of the major tourist attractions of Lakshadweep and there’s a lot of
concern related to this island and the government has taken a lot of initiatives for the very same.
• Coringa Wildlife Sanctuary is a wildlife sanctuary and estuary situated near Kakinada in Andhra
Pradesh, India. It is the second-largest stretch of mangrove forests in India with 24 mangrove tree
species and more than 120 bird species. It is home to the critically endangered white-backed vulture and
the long-billed vulture. In a mangrove ecosystem, the water bodies of the ocean/sea and the river meet
together at a certain point.
• Gahirmatha Marine Sanctuary is a marine wildlife sanctuary located in Odisha and is a very
popular tourist attraction of Odisha in India. It is the world's largest nesting beach for Olive Ridley
Turtles. It extends from Dhamra River mouth in the north to Brahmani river mouth in the south. It is very
famous for its nesting beach for olive ridley sea turtles. It is one of the world's most important nesting
beaches for turtles. Hence the correct option is (a)

Q 5.D
• The dynamic balance between the lithosphere and the asthenosphere is known as isostasy. This term
implies that the Earth’sgravity causes segments of the lithosphere to shift upward or downward according
to their densities. It is invoked to explain how different topographic heights can exist on the Earth's
surface.
• This essentially implies that wherever equilibrium exists on the Earth’s surface, the mass above the
surface will be balanced by an equal mass below it in the same are and the lighter crust floats on the
denser underlying mantle.
• There are two theories regarding the isostasy i.e. Pratt's theory and Airy's theory.
• At times, this balance gets disturbed due to violent earth movements happening inside the Earth and
tectonic disturbances which leads to natural disasters like Earthquake, volcanic eruption, and tsunami etc.
• The waxing and waning of ice sheets, erosion, mountain building, sedimentation are some other
processes that perturb isostasy.
• When large amounts of sediment are deposited in a particular region, the immense weight of the new
sediment may cause the crust below to sink. Similarly, when large amounts of material are eroded away
from a region, the land may rise to compensate. If a layer of ice is somehow sliced off the top of the
iceberg, the remaining iceberg will rise.
• The sediments that have collected are squeezed in the downfolds and fused into magma. The magma rises
to the surface through volcanic activity or intrusions of masses of magma as batholiths (massive rock
bodies). When the convection currents die out, the crust uplifts, and these thickened deposits rise and
become subject to erosion again.

Q 6.A
• The USA has recently announced sanctions against two officials of the International Criminal Court
(ICC).
• International Criminal Court is governed by an international treaty called 'The Rome Statute'. It is
the world's first permanent international criminal court. Hence statement 1 is correct.
• ICC headquarters at the Hague, the Netherlands. It investigates and where warranted, tries individuals
charged with the gravest crimes of concern to the international community like genocide, war
crimes,crimes against humanity and the crime of aggression.

2 www.visionias.in ©Vision IAS


• United Nations Security Council (UNSC) can also refer cases to ICC pursuant to a resolution adopted
under chapter VII of the UN charter. ICC is not a UN organization but it has a cooperation agreement
with the United Nations. Hence statement 2 is correct.
• India has neither signed nor ratified the Rome Statute. Many major countries, between them
constituting the majority of the world’s population, did not sign the Rome Statutes, including the United
States, Russia, China, India, Indonesia, and many Islamic countries, including Pakistan .111 states
have currently ratified the 1998 Rome Statutes, which entered into force on July 1, 2002 after ratification
by 60 countries.
• ICC judgements are binding, final and cannot be appealed. Hence statement 3 is not correct.

Q 7.C
• Manganese: Manganese is an important raw material for smelting of iron ore and also used for
manufacturing ferro alloys. Balaghat Mine is one of the largest mine producing one of the best quality of
manganese ore in the country. It is also the deepest underground manganese mine in Asia. About 80% of
the manganese production in India comes from Balaghat District. Hence, pair 4 is not correctly
matched.
• Aluminum: Bauxite is the ore that is used in the manufacturing of aluminum. Bauxite is found mainly in
tertiary deposits and is associated with laterite rocks occurring extensively either on the plateau or hill
ranges of peninsular India and also in the coastal tracts of the country.
o The state of Odisha is the largest producer of bauxite ore in India. More than 95% of the bauxite
resources of the state come under East Coast Bauxite. Most of the aluminum production comes from
the Panchpatmali hills mine operated by NALCO. Hence, pair 2 is not correctly matched.
o The patlands of Jharkhand in Lohardaga have rich deposits. Gujarat, Chhattisgarh, Madhya Pradesh
and Maharashtra are other major producers.
• Iron: India is endowed with fairly abundant resources of iron ore. It has the largest reserve of iron ore in
Asia. The two main types of ore found in our country are haematite and magnetite. The iron ore mines
occur in close proximity to the coal fields in the north-eastern plateau region of the country which adds to
their advantage.
o About 95 percent of total reserves of iron ore is located in the States of Odisha, Jharkhand,
Chhattisgarh, Karnataka, Goa, Telangana, Andhra Pradesh and Tamil Nadu. The important mines
are Gurumahisani, Sulaipet, Badampahar (Mayurbhaj), Kiruburu(Kendujhar) and Bonai
(Sundergarh). Hence, pair 3 is correctly matched.
• Copper: Copper is an indispensable metal in the electrical industry for making wires, electric motors,
transformers and generators. It is alloyable, malleable and ductile. It is also mixed with gold to provide
strength to jewelry. The major copper mines are the Khetri copper belt in Rajasthan, Singhbhum copper
belt in Jharkhand and Malanjkhand copper belt in Madhya Pradesh. Hence, pair 1 is correctly matched.

3 www.visionias.in ©Vision IAS


Q 8.B
• Skeletal soils refer to soils that contain 35 percent or more (by volume) of rock fragments, cobbles, gravel,
and laterite concretions or ironstones having diameters greater than 2 mm, within shallow depths (less
than 50 cm). Theses soils occur in a number of landscapes ranging from alluvial terraces, fans, erosional
surfaces, peneplains, hill slopes, and mountainous areas.
• Skeletal soils containing laterite at some period of time on erosional surfaces or as remnants of a
peneplain surface. Soils with cobblestones often develop on alluvial terraces. On the other hand, soils
shallow to bed rock or soils containing rock fragments are common on areas where the parent rocks are
subject to continuous erosion or weathering. They occupy mostly foothill slopes, mountains, and partial
peneplains or erosional surfaces.
• Due to a much different mode of soil formation, the skeletal soils are relatively variable in physical and
chemical properties. However, these soils are usually shallow, prone to erosion, and low in natural fertility
status.

4 www.visionias.in ©Vision IAS


Q 9.C
• Scientists from Agharkar Research Institute, Pune discover two new species of pipeworts from the
Western Ghats of Maharashtra & Karnataka. The one reported from Sindhudurg district of
Maharashtra was named as Eriocaulon parvicephalum (due to its minute inflorescence size), and the
other reported from Kumta, Karnataka was named as Eriocaulon karaavalense (named after
Karaavali = Coastal Karnataka region).
• The plant group known as pipeworts (Eriocaulon), whichcompletes their life cycle within a small period
during monsoon, exhibits great diversity in the Western Ghats, having around 111 species in India.
• Most of these are reported from the Western Ghats and Eastern Himalayas, and around 70% of them are
endemic to the country. One species, Eriocaulon cinereum, is well known for its anti-cancerous, analgesic,
anti-inflammatory, and astringent properties. E. quinquangulare is used against liver diseases. E.
madayiparense is an anti-bacterial from Kerala. The medicinal properties of the newly discovered species
are yet to be explored.
• In a separate study scientists at the Jawaharlal Nehru Tropical Botanic Garden and Research
Institute (JNTBGRI) at Palode here encountered the new species in the Akkamalai forest within the
Anamalai Tiger Reserve in Tamil Nadu recently. They have named it Ardisia ramaswamii after M.S.
Ramaswami. Ardisia ramaswamii (family Primulaceae) is a small tree which grows to a height of about
six meters.
• Since many related species of Ardisia possess medicinal properties, the discovery is significant from a
health-care perspective. For instance, Ardisia elliptica, known locally as ‘kili njaval’ or ‘sugar njaval,’ is
good for making wine and valued for treating diabetes. Another cousin, Ardisia solanacea,
(manimundakkizhangu), is used in traditional healing systems. Hence the correct answer is option (c)

Q 10.D
• The Hazardous Substances Management Division (HSMD) is the nodal point within the Ministry of
Environment, Forest and Climate Change for management of chemical emergencies and hazardous
substances.
• The main objective of the programme is to promote safe management and use of hazardous substances
including hazardous chemicals and hazardous wastes, in order to avoid damage to health and the
environment. The Division is also the nodal point for the following International Conventions/Agreements
viz.,
o Basel Convention on Control of Trans-boundary Movement of Hazardous Waste and their Disposal;
o Rotterdam Convention on Prior Informed Consent Procedure for certain Chemicals and Pesticides in
International Trade;
o Stockholm Convention on Persistent Organic Pollutants;
o Strategic Approach to International Chemicals Management; and
o Minamata Convention on Mercury.
• The Waigani Convention (Convention to Ban the Importation into Forum Island Countries of Hazardous
and Radioactive Wastes and to Control the Transboundary Movement and Management of Hazardous
Wastes within the South Pacific Region) - The main effect of this Convention is to ban the import of all
hazardous and radioactive wastes into South Pacific Forum Island Countries.
• Hence option (d) is the correct answer.

Q 11.B
• Confederation of Indian Industry's Green Building Congress 2020 was recently inaugurated.
• A green building is a building that, in its design, construction and operation, reduces or eliminates
negative impacts, and can create positive impacts, on our climate and natural environment.
• Some features which can make a building 'green ' include:
o Efficient use of energy, water, and other resources
o Use of renewable energy, such as solar energy
o Pollution and waste reduction measures, and the enabling of re-use and recycling
o Use of materials that are non-toxic and sustainable
• India has over 7.61 Billon Sq. Ft of green building footprint and amongst the top five countries in the
world.
• In India, there are predominantly three rating systems – Leadership in Energy and Environmental
Design (LEED), the rating systems from Indian Green Building Council (IGBC) and the Green Rating for
Integrated Habitat Assessment (GRIHA). In addition, there is also the Energy Consumption Building

5 www.visionias.in ©Vision IAS


Code (ECBC) and the National Building Code (NBC), which provide guidelines on energy consumption.
All buildings in India need to comply with these prescribed guidelines. Hence statement 1 is not correct.
• GRIHA is a five-star rating system for green buildings which emphasizes on passive solar
techniques for optimizing indoor visual and thermal comfort. The Energy and Resources Institute
(TERI) has developed GRIHA (Green Rating for Integrated Habitat Assessment), which was
adopted as the national rating system for green buildings by the Government of India.
o Endorsed by the Ministry of New and Renewable Energy, Government of India as of November
1, 2007, GRIHA has been developed to rate commercial, institutional and residential buildings in
India emphasizing national environmental concerns, regional climatic conditions, and indigenous
solutions.
o The rating applies to new building stock – commercial, institutional, and residential – of varied
functions. In order to address energy efficiency, GRIHA encourages optimization of building design
to reduce conventional energy demand and further optimize the energy performance of the building
within specified comfort limits. A building is assessed on its predicted performance over its entire life
cycle from inception through operation.
o GRIHA is a 100-point system consisting of some core points, which are mandatory, while the rest are
optional.
• Leadership in Energy and Environmental Design (LEED) is a rating system devised by the United
States Green Building Council (USGBC) to evaluate the environmental performance of a building.
It has been in use in India since 2001.
o LEED India is a green building rating system that helps to guide and design high-performance
commercial buildings in India. In an effort to strengthen consistency, the Green Building Certification
Institute (GBCI) now manages certification of projects to all LEED ratings systems in India, including
assuming responsibility for LEED India certification from the Indian Green Building Council (IGBC).
• The Indian Green Building Council (IGBC), part of the Confederation of Indian Industry (CII) was
formed in the year 2001. The vision of the council is, "To enable a sustainable built environment for all
and facilitate India to be one of the global leaders in the sustainable built environment by 2025".
o The council offers a wide array of services which include developing new green building rating
programmes, certification services and green building training programmes. Hence statement 2 is
correct.
o The council also organises Green Building Congress, its annual flagship event on green
buildings.
The council is committee-based, member-driven and consensus-focused. All the stakeholders of
construction industry comprising of architects, developers, product manufacturers, corporate,
Government, academia and nodal agencies participate in the council activities through local chapters. The
council also closely works with several State Governments, Central Government, World Green Building
Council, bilateral multi-lateral agencies in promoting green building concepts in the country.

Q 12.C
• The correct order of occurence of tribes from North to South is: 4-3-1-2. Hence option (c) is the correct
answer.
• The Pangwala is a tribal community predominant in the Pangi valley of Chamba district in Himachal
Pradesh. The official list of scheduled tribes in Himachal Pradesh includes a) Gaddis, b) Gujjars, c)
Kinnara or Kinnauras, d) Lahaulas and e) Pangwalas apart from some other smaller tribes groups like
Bhot/Bodh, Beda, Jad/Lamba/ Khampa, and Swangla.
• The Sahar, Sehariya, or Sahariya are an ethnic group in the state of Madhya Pradesh, India. The Saharias
are mainly found in the districts of Morena, Sheopur, Bhind, Gwalior, Datia, Shivpuri, Vidisha and Guna
districts of Madhya Pradesh and Baran district of Rajasthan.They are classified as Particularly vulnerable
tribal group.
• The Chenchus are Adivasi, a designated Scheduled Tribe in the Indian states of Andhra Pradesh,
Telangana, Karnataka and Odisha. They are an aboriginal tribe whose traditional way of life been based
on hunting and gathering. The Chenchus speak the Chenchu language, a member of the Dravidian
language family.
• The Shompen or Shom Pen are the indigenous people of the interior of Great Nicobar Island, part of the
Indian union territory of Andaman and Nicobar Islands. The Shompen are a designated Scheduled Tribe.
6 www.visionias.in ©Vision IAS
Q 13.A
• The correct order of the commodities in increasing order of their share in India's export basket is Iron and
Steel - Drug Formulations - Precious and Semi-Precious stones - Petroleum products. Hence option (a) is
the correct answer.

Q 14.B
• Salient features of Bio-Medical Waste Management (Amendment) Rules, 2018 are as follows:
o Bio-medical waste generators including hospitals, nursing homes, clinics, dispensaries, veterinary
institutions, animal houses, pathological laboratories, blood banks, health care facilities, and clinical
establishments will have to phase out chlorinated plastic bags (excluding blood bags) and gloves by
March 27, 2019.
o All healthcare facilities shall make available the annual report on its website within a period of two
years from the date of publication of the Bio-Medical Waste Management (Amendment) Rules, 2018.
o Operators of common bio-medical waste treatment and disposal facilities shall establish barcoding
and global positioning system for handling of bio-medical waste in accordance with guidelines issued
by the Central Pollution Control Board by March 27, 2019.
o The State Pollution Control Boards/ Pollution Control Committees have to compile, review and
analyze the information received and send this information to the Central Pollution Control Board in a
new Form (Form IV A), which seeks detailed information regarding district-wise bio-medical waste
generation, information on Health Care Facilities having captive treatment facilities, information on
common bio-medical waste treatment and disposal facilities.
o Every occupier, i.e. a person having administrative control over the institution and the premises
generating biomedical waste shall pre-treat the laboratory waste, microbiological waste, blood
samples, and blood bags through disinfection or sterilization on-site in the manner as prescribed by
the World Health Organization (WHO) or guidelines on the safe management of wastes from health
care activities and WHO Blue Book 2014 and then sent to the Common bio-medical waste treatment
facility for final disposal.
• Hazardous Waste Management Rules 2016
o For the first time, Rules have been made to distinguish between Hazardous Waste and other
wastes. Other wastes include: Waste tyres, paper waste, metal scrap, used electronic items, etc. and
are recognized as a resource for recycling and reuse. These resources supplement the industrial
processes and reduce the load on the virgin resource of the country.
• Plastic Waste Management Rules, 2016 aim to:
o Increase minimum thickness of plastic carry bags from 40 to 50 microns and stipulate minimum
thickness of 50 micron for plastic sheets also to facilitate collection and recycle of plastic waste
o Expand the jurisdiction of applicability from the municipal area to rural areas, because plastic has
reached rural areas also
7 www.visionias.in ©Vision IAS
o To bring in the responsibilities of producers and generators, both in plastic waste management system
and to introduce collect back system of plastic waste by the producers/brand owners, as per extended
producers responsibility
o To introduce collection of plastic waste management fee through pre-registration of the producers,
importers of plastic carry bags/multilayered packaging and vendors selling the same for establishing
the waste management system
o To promote use of plastic waste for road construction as per Indian Road Congress guidelines or
energy recovery, or waste to oil etc. for gainful utilization of waste and also address the waste
disposal issue; to entrust more responsibility on waste generators, namely payment of user charge as
prescribed by local authority, collection and handing over of waste by the institutional generator,
event organizers.
o An eco-friendly product, which is a complete substitute of the plastic in all uses, has not been found
till date. In the absence of a suitable alternative, it is impractical and undesirable to impose a blanket
ban on the use of plastic all over the country. The real challenge is to improve plastic waste
management systems.
• Hence option (b) is the correct answer.

Q 15.B
• Based on the Geological History of India, Rock System in India have been categorized as follows:
• The Archaean Rock System
o Archean System: These rocks are the oldest rocks formed about 4 billion years ago. They are formed
due to the solidification of molten magma. They are crystalline because they are volcanic in origin.
o Dharwar System: They are highly metamorphosed sedimentary rock-system formed between 4 – 1
billion years ago. They are abundant in the Dharwar district of Karnataka. They possess valuable
minerals like high-grade iron-ore, manganese, copper, lead, gold, etc.
• Purana Rock System (1400 – 600 Million Years)
o Cuddapah System: These are unfossiliferous clay, slates, sandstones and limestones that were
deposited in the depression between fold mountains. They are abundant in Cuddapah district of
Andhra Pradesh. These rocks contain ores of iron, manganese, copper, cobalt, nickel, etc.
o Vindhyan System: Deriving its name from the great Vindhyan mountains, it comprises of mostly
unfossiliferous ancient sedimentary rocks. It has diamond-bearing regions from which Panna and
Golconda diamonds have been mined.
• Dravidian Rock System (600 – 300 million years ago)
o These are fossil abundant rock system found in the Extra Peninsular region (Himalayas and Ganga
plain) and are very rare in Peninsular India.
o It comprises of rocks of Cambrian, Ordovician, Silurian, Devonian and Carboniferous periods,
however, Carboniferous rocks (350 million years) are most significant which comprise mainly of
limestone, shale and quartzite.
o Mount Everest is composed of Upper Carboniferous limestones. Coal formation started in the
Carboniferous age.
• Aryan Rock System (Upper Carboniferous to the Recent)
o Gondwana System: These are deposits laid down in synclinal troughs on ancient plateau surface and
derive its name from Gonds, the most primitive people of Telangana and Andhra Pradesh. Gondwana
rocks contain nearly 98 percent of India’s coal reserves. They have rich deposits of iron ore, copper,
uranium and antimony also.
o Deccan Trap: They are formed by volcanic outburst over a vast area of Peninsular India. These
volcanic deposits have flat top and steep sides and therefore called ‘trap’ meaning a ‘stair’ or ‘step’ in
Swedish. They have been weathered and eroded for millions of years to almost half of its original size
and has given birth to black cotton soil known as ‘regur’. It mainly occurs in parts of Kuchchh,
Saurashtra, Maharashtra, the Malwa plateau and northern Karnataka.
• Tertiary System
o It is about 60 to 7 million years ago.
o It is the most significant period in India’s geological history because the Himalayas were born and
India’s present form came into being in this period.
• Hence option (b), 2 - 3 - 4 - 1 is the correct chronological order.

8 www.visionias.in ©Vision IAS


Q 16.B
• The Bonn Challenge is a global effort to bring 150 million hectares of deforested and degraded land into
restoration by 2020 and 350 million hectares by 2030.
• It is an implementation vehicle for national priorities such as water and food security and rural
development while contributing to the achievement of international climate change, biodiversity, and land
degradation commitments.
• Launched by the Government of Germany and IUCN, the Bonn Challenge unites nations and regions
in bringing landscapes into restoration, to halt and reverse the effects of land degradation.
• Underlying the Bonn Challenge is the forest landscape restoration approach, which aims to restore
ecological integrity at the same time as improving human well-being through multifunctional
landscapes.
• The restoration of 150 million hectares of degraded and deforested lands in biomes around the world – in
line with the forest landscape restoration (FLR) approach – will create approximately US$ 84 billion
per year in net benefits that could bring direct additional income opportunities for rural communities.
About 90% of this value is potentially tradable, meaning that it encompasses market-related benefits.
Achieving the 350 million hectare goal will generate about US$ 170 billion per year in net benefits from
watershed protection, improved crop yields, and forest products, and could sequester up to 1.7
gigatonnes of carbon dioxide equivalent annually.
• The Bonn Challenge is not a new global commitment but rather a practical means of realizing many
existing international commitments, including the CBD Aichi Target 15, the UNFCCC REDD+ goal, and
the Rio+20 land degradation neutrality goal.
• Hence option (b) is the correct answer.

Q 17.D
• The government has brought "Over the Top" (OTT) platforms or video streaming service providers
under the ambit of Ministry of Information and Broadcasting.
• An "over-the-top" media service is any online content provider that offers streaming media as a
standalone product. The term is commonly applied to video-on-demand platforms, but also refers to
audio streaming, messaging services, or internet-based voice calling solutions. Hence statements 1, 2
and 3 are correct.
• Services circumvent traditional media distribution channels such as telecommunication or cable television
providers.
• So far in India, there are no laws or rules regulating OTT platforms as it is a relatively new medium of
entertainment. Unlike television, print or radio, which follow guidelines released by governments, OTT
platforms, classified as digital media or social media, had little to no regulation on the choice of content
they offered, the subscription rates, certification for adult movies and others.
• Earlier the Internet and Mobile Association of India (IAMAI), a representative body of the OTT
platforms and Online Curated Content Providers or OCCPs had proposed a self-regulatory model.
However, those were shot down by the Ministry of Information and Broadcasting.
• The government’s move to bring the OTT platforms under the I&B Ministry could also mean that in
future these platforms would have to apply for certification and approval of the content they wish to
stream.

Q 18.B
• ASEAN Defence Ministers' Meeting -Plus (ADMM-PLUS) meeting recently being hosted by Vietnam.
• In the 2nd ADMM held in Singapore in 2007 adopted a resolution to establish the ADMM-PLUS.
• The ADMM-PLUS is a platform for ASEAN and its eight dialogue partners to strengthen security
and defence cooperation for peace, stability, and development in the region.
• Eight Dialogue Partners are Australia, China, India, Japan, New Zealand, Republic of Korea, Russia,
and the USA (collectively referred to as the "Plus Countries").
• Hence option (b) is the correct answer.

Q 19.A
• Allelopathy refers to the chemical inhibition of one species by another.
• The 'inhibitory' chemical is released into the environment where it affects the development and growth of
neighboring plants.
• Allelopathic chemicals can be present in any part of the plant.
9 www.visionias.in ©Vision IAS
• They can be found in leaves, flowers, roots, fruits, or stems. They can also be found in the surrounding
soil. Target species are affected by these toxins in many different ways.
• The toxic chemicals may inhibit shoot/root growth, they may inhibit nutrient uptake, or they may attack a
naturally occurring symbiotic relationship thereby destroying the plant's usable source of a nutrient

Q 20.D

Q 21.B
• Biorock technology is an innovative process originally invented in 1976 by the late architect Professor
Wolf Hilbertz to produce natural building materials in the sea.
• Biorock materials are the only marine construction material that grows, get stronger with age, and are self-
repairing. Wolf called it Mineral Accretion Technology, Seacrete, or Segment. In 1987, biogeochemist
Dr Tom Goreau invited Wolf to come to Jamaica to work with him to develop applications for coral reef
restoration, and they decided to call the technology Biorock, because it not only grew hard limestone rock
for structural purposes, it greatly increased the growth of corals and all marine organisms. They founded
the Global Coral Reef Alliance to pursue research and development of Biorock Technology.
• Hence option (b) is correct.

Q 22.C
• Kumbh Mela, also called Kumbha Mela, in Hinduism, religious festival that is celebrated four times over
the course of 12 years, the site of the observance rotating between four pilgrimage places on four sacred
rivers—at Haridwar on the Ganges River, at Ujjain on the Shipra, at Nashik on the Godavari, and
at Prayag (modern Prayagraj) at the confluence of the Ganges, the Jamuna, and the mythical
Sarasvati.
• Each site’s celebration is based on a distinct set of astrological positions of the Sun, the Moon, and
Jupiter, the holiest time occurring at the exact moment when these positions are fully occupied. The
Kumbh Mela at Prayag, in particular, attracts millions of pilgrims. In addition, a Great Kumbh Mela
festival is held every 144 years at Prayag; the 2001 festival attracted some 60 million people.

10 www.visionias.in ©Vision IAS


Q 23.C
• The dynamic food-web model accounts for the bottom-up as well as top-down effects about the factors
contributing to the extinction risk of any particular species.
• The highest extinction risk is born by species with
o low energy input (e.g. species in high trophic levels receive the least energy), In the pyramid of
energy, the level of energy goes on decreasing as we move higher with every trophic level, hence the
energy available for the top-most predator will be significantly less and thus more susceptible to
extinction. Hence statement 1 is not correct.
o susceptibility to the loss of energy pathways (e.g. specialists with few prey species i.e species that
depend on a select few other species for their food) and
o dynamic instability (e.g. low Hill exponent and reliance on homogeneous energy channels when
feeding on similarly sized prey). Hence statement 2 and statement 3 are correct.
• Studies have found that the trophic level and not the body mass of a species influences its extinction risk.
On the other hand, body mass is the single most important factor determining the time to extinction of a
species, resulting in small species dying first. This suggests that in the field the trophic level might have
more influence on the extinction risk.

Q 24.B
• The streams within a drainage basin form certain patterns, depending on the slope of land, underlying rock
structure as well as the climatic conditions of the area.
• These are described as follows:
o Dendritic Drainage: The dendritic pattern develops where the river channel follows the slope of the
terrain. The stream with its tributaries resembles the branches of a tree, thus the name dendritic. A
pattern of drainage which is branching, ramifying or dichotomizing, thereby giving the appearance of
a tree. Most of the rivers of the Indo-Gangetic Plains are of dendritic type. Hence, pair 3 is
correctly matched.
o Trellis Drainage: A river joined by its tributaries, at approximately right angles, develops a trellis
pattern. A trellis drainage pattern develops where hard and soft rocks exist parallel to each other. In a
trellis pattern, the river forms a net-like system and the tributaries flow roughly parallel to each other.
The old folded mountains of the Chotanagpur Plateau have drainage of trellis pattern. Hence,
pair 4 is not correctly matched.
o Rectangular Drainage: It is marked by right-angled bends and right-angled junctions between
tributaries and the main stream. It differs from the trellis pattern in so far as it is more irregular and its
tributary streams are neither as long, nor parallel as in trellis drainage. A rectangular drainage pattern
develops on a strongly jointed rocky terrain. A typical example of this drainage pattern is found in
the Vindhyan Mountains of India. Hence, pair 1 is not correctly matched.
o Radial Pattern: The radial pattern develops when streams flow in different directions from a central
peak or dome-like structure. A good example of a radial drainage pattern is provided by the rivers
originating from the Amarkantak Mountain. Rivers like Narmada, Son and Mahanadi originating
from Amarkantak Hills flow in different directions. This pattern is also found in the Girnar Hills
(Kathiwar, Gujarat), and Mikir Hills of Assam. Hence, pair 2 is correctly matched.

Q 25.A
• The Government has launched The Economics of Ecosystems and Biodiversity TEEB-India
Initiative (TII) to highlight the economic consequences of the loss of biological diversity and the
associated decline in ecosystem services. Hence statement 1 is correct.
• The Initiative focussed on three ecosystems, namely forests, inland wetlands and coastal and marine
ecosystems.
• TII has been implemented under the Indo-German Biodiversity Programme as technical cooperation
with GIZ. Hence statement 2 is not correct.
• The outcome of the pilot projects will be fed into the sectoral synthesis for the three ecosystems. It was
released at the Brazil-India-Germany TEEB Dialogue, being hosted by India in September 2015. The
overall study report was released at the 21st session of the UNFCCC CoP held in November-December,
2015 in Paris.
• Pavan Sukhdev, environmental economist, and Gretchen C Daily, conservation biologist, shared
2020 ‘Nobel Prize for the Environment’. Pavan Sukhdev, an international banker by training, was the
lead expert on the first report of The Economics of Ecosystems and Biodiversity (TEEB) published in
2008 as an initiative the United Nations Environment Programme (UNEP).

11 www.visionias.in ©Vision IAS


Q 26.C
• Wealth Accounting and the Valuation of Ecosystem Services (WAVES) is a World Bank-led global
partnership that aims to promote sustainable development by ensuring that natural resources are
mainstreamed in development planning and national economic accounts. Hence the correct answer
is option (c)
• WAVES is now part of the broader World Bank umbrella initiative, the Global Program for
Sustainability (GPS).
• This global partnership brings together a broad coalition of UN agencies, governments, international
institutes, nongovernmental organizations and academics to implement Natural Capital Accounting
(NCA) where there are internationally agreed standards and develop approaches for other ecosystem
service accounts.
• By working with central banks and ministries of planning and finance across the world to integrate natural
resources into development planning through NCA, we hope to enable more informed decision making
that can ensure genuine green growth and long-term advances in wealth and human well-being.
• Objectives
o Help countries adopt and implement accounts that are relevant for policies and compile a body of
experience
o Develop approaches to ecosystem accounting methodology
o Establish a global platform for training and knowledge sharing
o Build international consensus around natural capital accounting
o Funding and Governance
• WAVES is funded by the European Commission, Denmark, France, Germany, Japan, the
Netherlands, Norway, Switzerland, and the United Kingdom.

Q 27.D
• Earth's rotation is the rotation of planet Earth around its own axis. Earth rotates eastward, in prograde
motion. As viewed from the north pole star Polaris, Earth turns counterclockwise.
• The Earth formed out of a nebula that collapsed. As the nebula collapsed it began rotating. The Earth's
rotation comes from the initial tendency to rotate that was imparted on it when it formed, only the
relatively weak tidal forces from the Moon act to slow it down.
• Melting land ice, like mountain glaciers and the Greenland and Antarctic ice sheets, will change the
Earth’s rotation only if the meltwater flows into the oceans. If the meltwater remains close to its source
(by being trapped in a glacier lake, for example), then there is no net movement of mass away from the
glacier or ice sheet, and the Earth’s rotation won’t change. But if the meltwater flows into the oceans and
is dispersed, then there is a net movement of mass and the Earth’s rotation will change. For example, if
the Greenland ice sheet were to completely melt and the meltwater were to completely flow into the
oceans, then global sea level would rise by about seven meters (23 feet) and the Earth would rotate
more slowly, with the length of the day becoming longer than it is today, by about two
milliseconds. Hence option 2 is correct.
• Because of Earth’s dynamic climate, winds and atmospheric pressure systems experience constant change.
These fluctuations may affect how our planet rotates on its axis. Changes in the atmosphere, specifically
atmospheric pressure around the world, and the motions of the winds that may be related to such climate
signals as El Niño are strong enough that their effect is observed in the Earth’s rotation signal.
• From year to year, winds and air pressure patterns change, causing different forces to act on the solid
Earth. During El Niño years, for example, the rotation of the Earth may slow because of stronger
winds, increasing the length of a day by a fraction of a millisecond (thousandth of a second). Hence
option 3 is correct.
• Earth's rotation is slowing slightly with time; thus, a day was shorter in the past. This is due to the
tidal effects the Moon has on Earth's rotation. As Earth rotates, the Moon's gravity causes the oceans to
seem to rise and fall. (The Sun also does this, but not as much.) There is a little bit of friction between the
tides and the turning Earth, causing the rotation to slow down just a little. Atomic clocks show that a
modern-day is longer by about 1.7 milliseconds than a century ago. Scientists predict that it would take 50
billion years for Earth to slow enough to permamently face the Moon, at which point the Moon would
stop receding from the Earth and Moon-induced tides on Earth will cease. So in short, yes, over very long
time periods the Moon will slow Earth's rotation to the point where there are no tides caused by the Moon,
but at that point the Earth-Moon system will no longer exist. Hence option 1 is correct.

12 www.visionias.in ©Vision IAS


• Other factors affecting rotational speed include the rapidity of a planet's initial formation (faster collapse
means more angular momentum conserved) and impacts from meteorites, which can slow down a planet
or knock it off stride.

Q 28.C
• Ramsar Convention on Wetlands is an intergovernmental treaty adopted in 1971 in the Iranian city
Ramsar, on the southern shore of the Caspian Sea.
• It came into force for India on 1st February 1982. Those wetlands which are of international importance
are declared as Ramsar sites.
• Recently Kabartal wetland (Bihar) and Asan Conservation Reserve (Uttarakhand), Lonar lake
(Maharashtra), Sur Sarovar /keetham lake (Agra) have been designated as Ramsar sites, making
them 'Wetland of International Importance'.
• With these 2 more inclusions, the total number of Ramsar sites in India is 41, the highest in South Asia.
• Rudrasagar lake in Tripura is a ramsar site. Hence Bihar and Tripura have Ramsar sites.
• Among the four South Indian states, Karnataka is the only state which has no Ramsar site. Hence
option 2 is correct.
• There are three Ramsar sites in eight states of northeast India which include Deepor Beel in
Assam, Loktak lake in Manipur and Rudrasagar in Tripura. There are no Ramsar sites in the
remaining North-East states of Arunachal Pradesh, Nagaland, Mizoram, Meghalaya and
Sikkim. Hence option 4 is correct.
• Some other notable Indian wetlands designated under the Ramsar convention are:
o Chilka (Odisha) and Keoladeo (Rajasthan) are the 1st wetland(s) designated.
o Sundarbans is the largest wetland in India and was designated in 2019.
o Smallest wetland is Renuka wetland in Himachal Pradesh

Q 29.C
• Biological Oxygen Demand (BOD):
o BOD is defined as the amount of oxygen demanded by the micro-organisms in the sewage for
the decomposition of bio-degradable matter under aerobic conditions. This is the most commonly
used parameter to determine the strength of the municipal or organic quality of the water i.e. it
measures biodegradable pollutants.
o The standard BOD test determines the amount of oxygen required by the micro-organisms for the
decomposition of the bio-degradable matter present in the wastewater sample under 5 days of the
aerobic condition at a temperature of 20 degree Celsius. It is measured in mg/l
o BOD is determined for designing treatment facilities. It is used to determine the size of activated
sludge units and trickling filter units. It is also used to determine the efficiency of various treatment
units.
• Chemical Oxygen Demand (COD):
o The amount of oxygen that is required for the chemical oxidation of the organic and inorganic
chemicals present in the wastewater by utilising oxidising agents like Potassium permanganate,
Potassium dichromate etc. is called as chemical oxygen demand (COD). COD measures both
biodegradable and non-biodegradable pollutants.
o The presence of COD facilitates rapid chemical oxidation of organic matter without any additional
equipment. This is the only method that enables to determine the organic load in heavy toxic sewage.
Hence it measures both biodegradable pollutants and non-biodegradable pollutants.
• The key difference between BOD and COD: BOD measures the amount of oxygen required by the aerobic
organisms to decompose organic matter and COD measures the oxygen required to decompose organic
and inorganic constituents present in the wastewater by chemical reaction. Hence, the value of COD is
greater than BOD.
• Hence, both statements are correct.

Q 30.C
• Statement 1 is correct: 'Bio' refers to living and 'remediate' means to fix or cure. It is a branch of
biotechnology that makes use of living organisms to reduce or eliminate environmental hazards resulting
from the accumulation of toxic chemicals, or hazardous waste. Both are bioremediation technologies.
Oilzapper is essentially a cocktail of five different bacterial strains that are immobilized and mixed with a
13 www.visionias.in ©Vision IAS
carrier material (powdered corncob). Oilzapper feeds on hydrocarbon compounds present in crude oil and
oily sludge (a hazardous hydrocarbon waste generated by oil refineries) and converts them into harmless
CO2 and water. Oilivorous-S a tad different from Oilzapper is an additional bacterial strain that makes the
former more effective against sludge and crude oil with high-sulphur content.
• Statement 2 is correct: Both Oilzapper and Oilivorous-S can be used in situ, thereby eliminating the
need to transfer large quantities of contaminated waste from the site, a process that poses more threats to
the environment.

Q 31.A
• Cattle population in India belongs to different breeds. These include:
• Milch breed: Here cows yield a higher quantity of milk but the bullocks are not of good quality. Some
important milch breeds are Gir, Sindhi, Sahiwal, Tharparkar and Deoni. The Gir breed yields about 3175
kg of milk per lactation period. Sahiwal breed yields about 2725-4535 kg of milk per lactation period.
• Draught Breeds: The cows are poor milkers but the bullocks are excelllent draught animals. This group
consists of Nagori, Bachaur, Kathiawar, Malvi Hallikar, Amritmahal, Kangyam, Killari, Ponwar and Siri.
• General Breeds: The cows are fairly good yielders of milk and the bullocks are good for draught
purposes. The group includes: e.g. Hariana, Ongale, Gaolo, Rath, Dangi, Nimari, Tharparkar and Kankrej.

Q 32.C
• Lake Effect snow occurs when cold air during the winters moves across the open waters of the lakes. As
the cold air passes over the unfrozen and relatively warm waters of the lakes, warmth and moisture are
transferred into the lowest portion of the atmosphere. The air rises, clouds form and grow into narrow
band that produces 2 to 3 inches of snow per hour or more. Hence, statement 1 is correct.
• Wind direction is a key component in determining which areas will receive lake effect snow. Heavy snow
may be falling in one location, while the sun may be shining just a mile or two away in either direction.
The physical geography of the land and water is also important.
• The areas affected by lake-effect snow are called snowbelts. These include areas east of the Great Lakes,
the west coasts of northern Japan, the Kamchatka Peninsula in Russia, and areas near the Great Salt Lake,
Black Sea, Caspian Sea, Baltic Sea, Adriatic Sea, and North Sea. Hence, statement 2 is correct.

Q 33.D
• Recent Context: Sansad Adarsh Gram Yojana (SAGY) completed 6 years. The Yojana was launched
in 2014, it is a village development project. Recently a performance review meeting of SAGY was done
via video conference wherein States were directed for diligent implementation of the scheme.
• The objective of the Yojana is to trigger processes that lead to holistic development of the identified
Gram Panchayats. It aimed to develop three Adarsh Grams by March 2019, of which one was to be
achieved by 2016. Thereafter, five such Adarsh Grams(one per year) will be selected and developed by
2024.
14 www.visionias.in ©Vision IAS
• MPs can select any gram panchayat, other than their own village or that of their spouse', to be
developed as an Adarsh Gram. The village must have a population of 3000-5000 people if it is located in
the plains, or 1000-3000 people if located in hilly areas. Hence statement 1 is not correct.
• Ministry of Rural Development will be the nodal Ministry coordinating and monitoring the scheme.
Hence statement 3 is not correct
• No new funds have been allocated for the yojana. Resources may be raised through funds from:
o existing schemes, such as Pradhan Mantri Gram Sadak Yojana, MGNREGA, etc.
o Members of Parliament Local Area Development Scheme(MPLADS),
o Gram panchayat's own revenue,
o Central and State Finance Commission Grants, and
o Corporate Social Responsibility funds. Hence statement 2 is not correct.

Q 34.B
• The Neem Campaign consisting of a group of NGOs and individuals, was initiated in 1993 in India.
This was done to mobilize worldwide support to protect indigenous knowledge systems and
resources of the Third World from piracy by the West, particularly in light of emerging threats
from intellectual property rights regimes under WTO and TRIPS. The Neem patent became the first
case to challenge European and US patents on grounds of biopiracy.
• The European Patent Office (EPO) delivered a favourable interim judgement on the challenge of a
European patent on the fungicidal effects of neem oil (Patent No. 436 257 B1) owned by W. R. Grace & Co.
• The European Patents Office accepted the arguments offered by Indian scientists and rejected the order of
the US patents office to award the patent to W R Grace, a US-based company, at the last hearing of the
case.
• The Indian scientists argued that the people of India have known the medicinal properties of neem for
thousands of years and hence no other company can patent its properties. The EPO accepted the
argument. Hence the correct answer is option (b)

Q 35.C
• Global Hunger Index(GHI) has been released recently by Concern Worldwide and Welthungerhilfe
which ranked India on the 94th spot among 107. countries. National Family Health Survey(NFHS)in
2015-16 shows some parameters of for child malnutrition like 'wasted', 'stunted' on which much needs to
be done.
o Stunting refers to low height for age. Hence pair 1 is not correctly matched.
o Wasting refers to low weight for height. Hence pair 2 is not correct correctly matched.
o Undeweight refers to low weight for age. Hence pair 3 is correctly matched.
• As per NFHS-4 (2015-16), 35.7 per cent children below five years are underweight, 38.4 per cent are
stunted and 21 per cent are wasted in the country.
• Central governments have initiated various programs like MAA (Mothers Absolute Affection) to
promote exclusive breast feeding;
• Pradhan Mantri Surakshit Matritva Abhiyan; and other initiatives to combat the challenges of
malnutrition in the country like:
• Facility Based Management of Children with Severe Acute Malnutrition (SAM).
• India Newborn Action Plan (INAP) .
• Enhancing optimal Infant and Young Child Feeding (IYCF) practices.
• Home Based Newborn Care (HBNC) and Facility Based Newborn Care (FBNC).
• National Iron Plus Initiative (NIPI) for anaemia control among children from 6 to 60 months of age.

Q 36.D
• A bioindicator is a living organism that gives us an idea of the health of an ecosystem. Some organisms
are very sensitive to pollution in their environment, so if pollutants are present, the organism may change
its morphology physiology or behaviour, or it could even die.
• Bioindicators can be plants, animals, or microorganisms:
o If toxins are present, certain plants may not be able to grow in the area affected.
15 www.visionias.in ©Vision IAS
o Monitoring population numbers of animals may indicate damage to the ecosystem in which they live.
o Algae blooms are often used to indicate large increases of nitrates and phosphates in lakes and rivers.
• Hence option (d) is correct.

Q 37.C
• Stockholm Convention is an international environmental treaty, signed in 2001 and effective from May
2004, that aims to eliminate or restrict the production and use of persistent organic pollutants (POPs).
Hence pair 1 is correctly matched.
• [Persistent organic pollutants (POPs), sometimes known as "forever chemicals" are organic compounds
that are resistant to environmental degradation through chemical, biological, and photolytic processes.]
• Basel Convention is an international treaty that was designed to reduce (not ban) the movements
of hazardous waste between nations, and specifically to prevent the transfer of hazardous waste from
developed to less developed countries (LDCs). It does not, however, address the movement of
radioactive waste. The convention is also intended to minimize the amount and toxicity of wastes
generated, to ensure their environmentally sound management as closely as possible to the source of
generation, and to assist LDCs in the environmentally sound management of the hazardous and other
wastes they generate. Hence pair 2 is not correctly matched.
• Rotterdam Convention is a multilateral treaty to promote shared responsibilities in relation to the
importation of hazardous chemicals. The convention promotes the open exchange of information and
calls on exporters of hazardous chemicals to use proper labeling, include directions on safe handling, and
inform purchasers of any known restrictions or bans. Signatory nations can decide whether to allow or ban
the importation of chemicals listed in the treaty, and exporting countries are obliged to make sure that
producers within their jurisdiction comply. Hence pair 3 is correctly matched.

Q 38.D
• The Wildlife Protection Act of 1972 was amended in 2006 to provide for constituting the National
Tiger Conservation Authority responsible for the implementation of the Project Tiger plan to protect
endangered tigers. The National Tiger Conservation Authority is set up under the Chairmanship of the
Minister for Environment and Forests.
• Central Zoo Authority: Section 38A of WPA: The Central Government shall constitute a body to be
known as the Central Zoo Authority to exercise the powers conferred on and to perform the functions
assigned to it under this Act.
• The Central Government shall, within three months from the date of commencement of the Wild Life
(Protection) Amendment Act, 2002 (16 of 2003), constitute the National Board for Wild Life with
the Prime Minister as Chairperson.
• The State Government shall constitute a State Board for Wild Life with Chief Minister of the
State and in case of the Union territory, either Chief Minister or Administrator, as the case may be -
Chairperson;
• The National Biodiversity Authority (NBA) is a statutory autonomous body under the Ministry of
Environment, Forests and climate change, Government of India established in 2003 to implement the
provisions under the Biological Diversity Act, 2002, after India signed Convention on Biological
Diversity (CBD) in 1992.
• Hence option (d) is the correct answer.

Q 39.B
• Recent Context: Pilibhit Tiger Reserve (PTR) in Uttar Pradesh has bagged the first international
award, TX2, among the 13 tiger ranging countries for having doubled the number of tigers in less than
the stipulated time.
• TX2 is the global award which was set up in 2010 in St. Petersburg, Russia by international organisations
working for tiger conservation like WWF, UNDP, IUCN, Global Tiger Fund (GTF).
• The TX2 goal is a global commitment to double the world's wild tigers by 2022. The goal has
been set by the World Wildlife Fund (WWF) through the Global Tiger Initiative and Global Tiger
Forum. Hence statement 1 is not correct
• Nepal became the first country in the world to double its tiger population in a decade as part of WWF
'TX2' programme which aims to double the number of tigers by 2022. Hence statement 2 is correct.

16 www.visionias.in ©Vision IAS


• Tiger range countries: India, Bangladesh, Bhutan, Cambodia, China, Indonesia, Lao PDR, Malaysia,
Myanmar, Nepal, Russia, Thailand and Vietnam.
• Tiger is categorised as 'Endangered' under IUCN Red List and listed under Appendix I of CITES.
India's Project Tiger was launched in 1973 with 9 tiger reserves. Tiger has been listed under 'Schedule I'
of the Wildlife Protection Act,1972.
• As per 2018 Tiger Census, India is home to 2,967 tigers, which is about 75% of the global tiger
population.

Q 40.C
• Jet streams are relatively narrow bands of strong wind in the upper levels of the atmosphere. The winds
blow from west to east in jet streams but the flow often shifts to the north and south. Jet streams follow
the boundaries between hot and cold air.
• Since these hot and cold air boundaries are most pronounced in winter, jet streams are the strongest for
both the northern and southern hemisphere winters. Hence, statement 1 is correct.
• The actual appearance of jet streams results from the complex interaction between many variables - such
as the location of high and low-pressure systems, warm and cold air, and seasonal changes. They meander
around the globe, dipping and rising in altitude/latitude, splitting at times and forming eddies, and even
disappearing altogether to appear somewhere else.
• Jet streams are often indicated by a line on a weather map indicating the location of the strongest wind.
However, jet streams are wider and not as distinct as a single line but are actually regions where the wind
speed increases toward a central core of greatest strength. Hence, statement 2 is not correct.
• One way of visualizing this is to consider a river. The river's current is generally the strongest in the
center with decreasing strength as one approaches the river's bank. Therefore, it is said that jet streams are
"rivers of air".
• Jet streams also "follow the sun" in that as the sun's elevation increases each day in the spring, the average
latitude of the jet stream shifts poleward. With summer approaching in Northern Hemisphere, jet streams
gradually keep shifting northwards. As Autumn approaches and the sun's elevation decreases, the jet
stream's average latitude moves toward the equator. Hence, statement 3 is correct.

Q 41.A
• India recently refused to become a signatory to the Osaka declaration on digital economy which
proposes the concept of Data Free Flow with Trust (DFFT). Hence option (a) is the correct answer.
• DFFT aims to eliminate restrictions on cross border transfer of information by electronic means,
including personal information and storing data in foreign servers for productivity, innovation and
sustainable development.
• It also stresses on the importance of addressing challenges such as security, data protection and
intellectual property.
• Brazil declaration is about adopting a road map by UNHCR to address new displacement trends and
end statelessness by 2024.
• Montreal declaration is about responsible development of Artificial Intelligence.
• Stockholm declaration is about the Human Environment adopted in 1972 by United Nations
Conference.

Q 42.B
• Recent Context: India and China are working out modalities to finalise a plan for a phased
disengagement and de-escalation along the LINE OF ACTUAL CONTROL (LAC) in eastern Ladakh.
The Line of Actual Control separates Indian and Chinese troops since 1962 generally runs along with the
land except for the width of Pangong Tso. India controls about 45 km stretch of the Pangong Tso and
china the rest.
• Geography of Pangong Tso:
o It is a long narrow, deep, endorheic (landlocked) lake situated at a height of more than 14,000 ft in
the Ladakh Himalayas. Hence statement 2 is correct.
o Pangong Tso is not a part of the Indus river basin area. Hence statement 1 is not correct.
o The brackish water lake freezes over in winter and becomes ideal for ice skating and polo. Hence
statement 3 is not correct.
o An inner line permit is required to visit the lake as it lies on the Sino-India Line of Actual Control.

17 www.visionias.in ©Vision IAS


• Tactical significance of the lake:
o It lies in the path of the Chushul approach, one of the main approaches that China can use for an
offensive into Indian-held territory.
o Indian assessments show that a major Chinese offensive if it comes, will flow across both the north
and south of the lake.
o During the 1962 war, this was where China launched its main offensive — the Indian Army fought
heroically at Rezang La, the mountain pass on the southeastern approach to Chushul valley.

Q 43.B
• The Nilgiri tahr is an ungulate that is endemic to the Nilgiri Hills and the southern portion of the
Western & Eastern Ghats in the states of Tamil Nadu and Kerala in Southern India. It is the state
animal of Tamil Nadu. It is listed as Endangered in the IUCN Red List.
• The Nilgiri Tahr can be found only in India. It inhabits the open montane grassland habitat of the South
Western Ghats montane rain forests ecoregion. At elevations from 1,200 to 2,600 m (3,900 to 8,500 ft),
the forests open into large grasslands interspersed with pockets of stunted forests, locally known
as sholas. These grassland habitats are surrounded by dense forests at the lower elevations.
• Their range extends over 400 km (250 mi) from north to south, and Eravikulam National Park is
home to the largest population. As per the wildlife census conducted by the Kerala forest department in
association with volunteers from the College of Forestry and Veterinary Science under Kerala
Agricultural University, from April 24–28, 2014, the number of animals in Eravikulam National Park has
increased to 894 individuals. This is the highest ever count recorded in the national park, with the first
census in 1996 finding only 640 tahrs.
• The other significant concentration is in the Nilgiri Hills, with smaller populations in the Anamalai
Hills, Periyar National Park, Palani Hills, and other pockets in the Western Ghats south of Eravikulam,
almost to India's southern tip. A small population of tahrs numbering around 200 is known to inhabit the
Boothapandi, Azhakiyapandipuram, Velimalai, Kulasekaram, and Kaliyal Ranges in the Kanyakumari
district of Tamil Nadu and another small herd of less than 30 animals is known to inhibit Ponmudi Hills
in Trivandrum district of Kerala.
• The Kishanpur Wildlife Sanctuary is a part of the Dudhwa Tiger Reserve near Mailani in Uttar Pradesh,
India.
• The term ‘Askot’ is derived from 'Assi Kot' or Eighty Forts, many of which are located in Nepal.
• Askot Wildlife Sanctuary is one of India’s most famous wildlife sanctuaries and is rightly known as the
‘green paradise of India. Askot Wildlife Sanctuary, with an altitude range from 600 m (2,000 ft) to 6,905
m (22,654 ft) is located in the Pithoragarh district of Kumaun. It is well known for musk deer. Hence the
correct answer is option (b)

18 www.visionias.in ©Vision IAS


Q 44.B
• The Malabar large-spotted civet (Viverra civettina), also known as the Malabar civet, is a viverrid
endemic to the Western Ghats of India. It is listed as Critically Endangered on the IUCN Red List as
the population is estimated to number fewer than 250 mature individuals.
• It has not been recorded during surveys carried out between 1990 and 2014. In the early 1990s, isolated
populations still survived in less disturbed areas of South Malabar but were seriously threatened by habitat
destruction and hunting outside protected areas.
• Until a few decades ago, local merchants in Kerala reared Malabar civets to obtain civetone, an extract
from the scent gland, which was used in medicine, and as an aromatic.
• It is now seriously threatened by habitat destruction and fragmentation. Until the 1990s, it was confined to
remnant forests and disturbed thickets in cashew and rubber plantations in northern Kerala, where the
hunting pressure was another major threat.
• The Malabar civet is considered nocturnal and so elusive that little is known about its biology and
ecology apart from habitat use. Hence the correct answer is option (b).

Q 45.B
• The Central Pollution Control Board of India (CPCB) is a statutory organisation under the
Ministry of Environment, Forest and Climate Change. It was established in 1974 under the Water
Act, 1974. The CPCB is also entrusted with the powers and functions under the Air Act, 1981. Hence,
statement 1 is not correct.
• The CPCB has wide-ranging powers and responsibilities like:
o To advise the central government on any matter related to the prevention and control of water
and air pollution and improvement of the quality of air. Hence, statement 2 is correct.
o Improvement of air quality
o Plan nationwide programs for the prevention
o Control and abatement of water and air pollution
o Coordinate the activities of SPCBs and resolve disputes among them.

Q 46.D
• A seamount is an underwater mountain with steep sides rising from the seafloor. Seamounts are often
remnants of extinct volcanoes and come in a variety of shapes and sizes. By definition, seamounts are
geological structures that rise more than 1,000 meters (3,300 feet) above the surrounding seafloor, but
most are much taller than that, rising upwards of 5,000 meters (16,400 feet), sometimes to within a few
hundred meters of the ocean surface. Seamounts can be found in every world ocean basin.
• Studies conducted over seamounts indicate that seamounts function as “oases of life,” with higher species
diversity and biomass found on the seamount and in the waters around it than on the flat seafloor.
• Seamounts rise up high in the water column, creating complex current patterns influencing what lives on
and above them. Seamounts also provide substrate (a location for attachment) where organisms can settle
and grow. These organisms provide a food source for other animals. Many deep-sea animals, such as
corals and sponges, are sessile, meaning they spend most of their lives permanently attached to rocks. As
opposed to a flat seafloor, seamounts rise off the ocean bottom and interact with water flowing around
them. These water currents can wash off sediment on a seamount, exposing rocks that are ideal habitat for
animals that require hard substrate to grow and attach. Hence option 1 is correct.
• Also, because these animals cannot move around to find food, they are dependent upon ocean currents to
bring their food to them. The strong currents running over a seamount can deliver animals living along its
flanks with a constant supply of planktonic food. Hence option 2 is correct.
• These currents also produce localized upwelling of water around the seamount. Nutrients like nitrates and
phosphates, which are critical to the growth of phytoplankton, are lifted from the deep to the sunlit surface
waters. These nutrients fuel an explosion of planktonic plant and animal growth and attract larger animals
such as whales, sharks, tunas, and seabirds to a veritable feast. Hence option 3 is correct.
• Additionally, those same currents carry larvae, like shipwrecked mariners, from various geographically
distant areas. Seamounts thus provide a place for these lost larvae to settle and grow, so animals that may
normally be not found together can be found on seamounts living side by side.

19 www.visionias.in ©Vision IAS


Q 47.B
• The Blue Carbon Initiative is a global program working to mitigate climate change through the
restoration and sustainable use of coastal and marine ecosystems. The Initiative currently focuses on
mangroves, tidal marshes, and seagrasses. The Blue Carbon Initiative brings together governments,
research institutions, non-governmental organizations, and communities from around the world. The
Initiative is coordinated by
o Conservation International (CI),
o the International Union for Conservation of Nature (IUCN),
o the Intergovernmental Oceanographic Commission of the United Nations Educational, Scientific, and
Cultural Organization (IOC-UNESCO).
• Hence option (b) is the correct answer.

Q 48.D
• Wetlands are areas where the water table is at or near the surface level, or the land is covered by shallow
water. The Ramsar Convention defines wetlands as: “areas of marsh, fen, peatland or water,
whether natural or artificial, permanent or temporary, with water that is static or flowing, fresh,
brackish or salt, including areas of marine water the depth of which at low tide does not exceed six
meters”.
• Moreover wetlands “may incorporate riparian and coastal zones adjacent to the wetlands, and islands or
bodies of marine water deeper than six meters at low tide lying within the wetlands”.
• The Ramsar Classification of Wetland Types includes 42 types of wetlands, which belong to one of
the three broad categories (Ramsar Convention Secretariat, 2011):
o Inland wetlands;
o Marine/coastal wetlands;
o Human-made wetlands. Hence the correct answer is option (d)
• Human-made wetlands covered by the Ramsar Convention include aquacultures, farm ponds, and
permanently or temporarily inundated agricultural land- such as rice paddies, salt pans, reservoirs,
gravel pits, sewage farms and canals.
• There is a range of other wetland classifications used for different purposes, based on
hydrogeomorphology and/or vegetation characteristics, such as :
o Marine (coastal wetlands, including coastal lagoons, rocky shores and coral reefs);
o Estuarine (including deltas, tidal marshes, and mangrove swamps);
o Lacustrine (wetlands associated with lakes);
o Riverine (rivers and wetlands along rivers and streams); and
o Palustrine (marshes, swamps and bogs).

Q 49.D
• Photosynthesis takes place only in the green parts of the plants, mainly the leaves, however, it can
occur in the stems, but in a minimal proportion.
• Plant roots usually grow underground as heterotrophic organs and depend on aerial leaves for
energy, although the roots of some epiphytic plants turn green and perform active photosynthesis.
Example: Arabidopsis is the angiosperms dicot plant that develops the chloroplast in roots when detached
from the shoot and accumulates chlorophyll. Hence the correct option is (d)
• Within the leaves, the mesophyll cells have a large number of chloroplasts that are responsible for CO2
fixation. Within the chloroplasts, the membranes are sites for the light reaction, while the chemosynthetic
pathway occurs in the stroma.
• Photosynthesis has two stages: the light reaction and the carbon fixing reactions.
o The light dependant reaction (light is required) takes place in the thylakoids and converts light into
chemical energy. The chemical energy produced in the light-dependent reaction is then used to fix
CO2 into carbohydrates, which is the light-independent reaction, or Calvin cycle. The Calvin cycle
occurs in the stroma of the chloroplast. The by-products of these reactions are glucose, which is used
by the plant, and oxygen, which is released to the atmosphere through the stomata.
o Plants differ in the pathway to fix CO2; as a consequence, the light-independent reactions can be
classified as pathway C3, pathway C4 and pathway CAM. C3 plants are adapted to temperate climates
while C4 plants are adapted to hot, dry climates. The CAM plants are mostly cacti; they open the
stomata at night to reduce the loss of water by transpiration.
20 www.visionias.in ©Vision IAS
Q 50.C
• The expansion of the universe is the increase in distance between any two given gravitationally unbound
parts of the observable universe with time.
• It is an intrinsic expansion whereby the scale of space itself changes. The universe does not expand "into"
anything and does not require space to exist "outside" it. Technically, neither space nor objects in space
move. To an observer, it appears that space is expanding and all but the nearest galaxies are receding into
the distance.
• It is specifically the vast regions of space between galaxies which is expanding. Hence only option 3 is
correct.
• The galaxies sit more or less passively in the space around them. As the space between galaxies
expands, it carries the galaxies further apart - like raisins in an expanding dough.
• The expansion of the Universe was inferred from early twentieth-century astronomical observations and is
an essential ingredient of the Big Bang theory.

Q 51.A
• Lakes and ponds are divided into three different "zones" which are usually determined by depth and
distance from the shoreline.
• Littoral Zone- The topmost zone near the shore of a lake or pond is the Littoral zone. This zone is the
warmest since it is shallow and can absorb more of the Sun's heat. It sustains a fairly diverse community,
which can include several species of algae (like diatoms), rooted and floating aquatic plants, grazing
snails, clams, insects, crustaceans, fishes, and amphibians. The vegetation and animals living in the littoral
zone are food for other creatures such as turtles, snakes and ducks.
• Limnetic Zone- The near-surface open water surrounded by the littoral zone is the limnetic zone. The
limnetic zone is the open water area where light does not penetrate to the bottom. The limnetic zone is
well-lighted (like the littoral zone) and is dominated by plankton, both phytoplankton and
zooplankton. Plankton is small organisms that play a crucial role in the food chain. Without aquatic
plankton, there would be few living organisms in the world, and certainly no humans. A variety of
freshwater fish also occupies this zone.
• Profundal Zone- The deep-water part of the lake or pond is called the profundal zone. This zone is much
colder and denser than the other two. Little light penetrates all the way through the limnetic zone into the
zone. The fauna is heterotrophs that eat dead organisms and use oxygen for cellular respiration.
• Hence, option (a) is correct.

Q 52.D
• Recently, the Public Accounts Committee submitted its report on the Accelerated Irrigation
Benefits Programme (AIBP).
• The Accelerated Irrigation Benefits Programme was launched during 1996-97 to provide loan
assistance to poor farmers. The Command Area Development Programme was launched in 1974-
75 for the development of water use efficiency. Hence statement 1 is not correct.
• After launch of Pradhan Mantri Krishi Sinchai Yojana (PMKSY) in 2015-16, AIBP became a part of
PMKSY. It is being implemented by Ministry of Jal Shakti. Hence statement 2 is not correct.
• Accelerated Irrigation Benefit Programme also aimed to provide financial assistance to State
governments with the aim of speeding up the implementation of on-going irrigation projects. These
included:
o Major/Medium Irrigation (MMI) including Extension, Renovation, and Modernization (ERM) of
irrigation projects
o Surface Minor Irrigation schemes (SMI)
o Lift Irrigation Schemes (LIS)

Q 53.C
• Amazon River
o It flows in South America. It is the largest river by discharge volume of water in the world.
o The Mantaro and Apurímac rivers join, and with other tributaries form the Ucayali River, which in
turn meets the Marañón River upstream of Iquitos, Peru, they form what countries other than Brazil
consider to be the main stem of the Amazon.
o Brazilians call this section the Solimões River above its confluence with the Rio Negro forming what
Brazilians call the Amazon at the Meeting of Waters at Manaus, the largest city on the river.

21 www.visionias.in ©Vision IAS


• Zambezi River
o It is the longest east-flowing river in Africa and the largest flowing into the Indian Ocean from Africa.
o It arises in Zambia and flows through eastern Angola, along the north-eastern border of Namibia and
the northern border of Botswana, then along the border between Zambia and Zimbabwe to
Mozambique, where it crosses the country to empty into the Indian Ocean.
o The Zambezi's most noted feature is Victoria Falls.
• Mississippi River
o It is the second-longest river and chief river of the second-largest drainage system on the North
American continent, second only to the Hudson Bay drainage system.
o It rises from Lake Itasca in Minnesota and flows south to form delta in the Gulf of Mexico.
• Narmada River
o It rises from Amarkantak Plateau in Madhya Pradesh.
o It flows westwards before draining through the Gulf of Khambhat into the Arabian Sea.
o It is one of the rivers in India that flows in a rift valley, flowing west between the Satpura and
Vindhya ranges.

Q 54.A
• Sustainable Alternative Towards Affordable Transportation (SATAT) is an initiative aimed at setting
up Compressed Bio-Gas production plants and make it available in the market for use in automotive
fuels by inviting Expression of Interest from potential entrepreneurs.
• The initiative was launched in October 2018 by the Ministry of Petroleum & Natural Gas in association
with Public Sector Undertaking (PSU) Oil Marketing Companies (OMC) viz. Indian Oil Corporation Ltd.,
Bharat Petroleum Corporation Ltd., and Hindustan Petroleum Corporation Ltd.
• The government of India, under the SATAT initiatives, envisages setting up of 5000 CBG plants by 2023-
24 with a production target of 15 MMT, facilitating the creation of new employment opportunities and
enhancing farmers' income towards further invigorating the rural economy.
• Hence option (a) is correct.

Q 55.A
• Eco-Sensitive Zones or Ecologically Fragile Areas are areas within 10 km around Protected Areas,
National Parks and Wildlife Sanctuaries. ESZs are notified by MoEFCC, Government of India under
the Environment Protection Act 1986. Hence, statement 1 is correct.
• The basic aim is to regulate certain activities around National Parks and Wildlife Sanctuaries so as to
minimise the negative impacts of such activities on the fragile ecosystem encompassing the protected
areas. ESZs would ensure that these areas act as “shock absorbers” to the protected areas by
regulating and managing the activities around such areas. Hence, statement 2 is correct.
• They also act as a transition zone from areas of high protection to areas involving lesser protection. ESZs
help in in-situ conservation, which deals with conservation of an endangered species in its natural habitat,
for example the conservation of the One-horned Rhino of Kaziranga National Park, Assam.
• As per MoEFCC guidelines broad-based thematic activities have been classified as prohibited, restricted
with safeguards and permissible. Activities like commercial mining, setting of sawmills and industries
causing pollution etc. are prohibited. Felling of trees, drastic change in agriculture systems, setting up
of hotels and resorts etc. are regulated. Activities permitted in the areas include ongoing agriculture and
horticulture practices by local communities, rainwater harvesting etc. Recently the Jammu and Kashmir
government set up a ten-member committee to declare Srinagar’s famous Dal Lake and its surrounding
areas as an eco-sensitive zone (ESZ). Hence, statement 3 is not correct.
22 www.visionias.in ©Vision IAS
Q 56.A
• Region to the South of China and the north of India is known as the “Third Pole”. The region
encompasses the Himalaya-Hindu Kush mountain ranges and the Tibetan Plateau. Hence option (a)
is the correct answer.
• The region is world’s third largest store of ice after Antarctica and Greenland that’s roughly the size of
Texas and New Mexico combined.
• It is estimated that the water that flows from the Third Pole supports 120 million people directly through
irrigation systems, and a total of 1.3 billion indirectly through river basins in China, India, Nepal,
Pakistan, Bangladesh and Afghanistan. 10 of Asia’s largest rivers begin here, including the Yellow river
and Yangtze river in China, the Irrawaddy river in Myanmar, the Ganges, which flows through India and
Bangladesh, and the trans-boundary Mekong river.
Q 57.C
• Sea Ice and Land Ice
• The most basic difference is that sea ice forms from salty ocean water, whereas icebergs, glaciers, and
lake ice form from freshwater or snow. Sea ice grows, forms, and melts strictly in the ocean.
• Glaciers are considered land ice, and icebergs are chunks of ice that break off of glaciers and fall into the
ocean. Lake ice is made from fresh water and freezes as a smooth layer, unlike sea ice, which develops
into various forms and shapes because of the constant turbulence of ocean water.
• The process by which sea ice forms is also different from that of the lake or river ice. Freshwater is unlike
most substances because it becomes less dense as it nears the freezing point. This difference in density
explains why ice cubes float in a glass of water. Very cold, low-density freshwater stays at the surface of
lakes and rivers, forming an ice layer on the top.
• In contrast to freshwater, the salt in ocean water causes the density of the water to increase as it nears the
freezing point, and very cold ocean water tends to sink. As a result, sea ice forms slowly, compared to
freshwater ice, because saltwater sinks away from the cold surface before it cools enough to
freeze. Hence, statement 1 is correct.
• Furthermore, other factors cause the formation of sea ice to be a slow process. The freezing temperature
of saltwater is lower than freshwater; ocean temperatures must reach -1.8 degrees Celsius to freeze.
Because oceans are so deep, it takes longer to reach the freezing point, and generally, the top 100 to 150
meters (300 to 450 feet) of water must be cooled to the freezing temperature for ice to form.
• Icebergs and frozen seawater also melt in warm temperatures but do not cause sea levels to rise. This is
because they are already in the water. The volume of water they displace as ice is the same as the volume
of water they add to the ocean when they melt. As a result, the sea level does not rise when sea ice
melts. Hence, statement 2 is correct.
• Sea level is rising, in part, because melting glaciers on land are adding more water to Earth’s oceans. As
temperatures rise, glaciers melt faster than they accumulate new snow. As these ice sheets and glaciers
melt, the water eventually runs into the ocean, causing sea level to rise.

Q 58.D
• Recently a newly constituted Panda committee under the Tribal Affairs Ministry is to look into the
issue of forest rights of the Mankidia tribe in Simlipal Tiger Reserve. Mankidiya is one of the
13 Particularly Vulnerable Tribal Groups (PVTG) in Odisha. They critically depend on making rope
with siali fibre that is richly available in Similipal Tiger Reserve (STR). They were denied habitat rights
inside STR under Scheduled Tribes and Other Traditional Forest Dwellers (Recognition of Forest
Rights) Act 2006. Hence pair 1 is correctly matched.
• The Jharkhand government has passed a resolution to send the center a letter to recognize Sarna
religion and include it as a separate code in the census of 2021. The followers of Sarna faith believe in
praying to nature. The motto of the faith is "Jal, Jungle, Zameen". Hence pair 2 is correctly matched.
• The Uttar Pradesh government has recently embarked upon a scheme to take the unique culture of its
ethnic Tharu tribe across the world. Most of them are forest dwellers and some practiced agriculture.
They live mostly in Uttar Pradesh, Uttarakhand, and Bihar. They are the followers of Theravada
Buddhism. Hence pair 3 is correctly matched.

Q 59.D
• Damodar River basin is a part of the Ganges River System. It flows through the states of Jharkhand West
Bengal states. Various industrial towns are located in the adjoining regions such as Bokaro, Dhanbad,
Asansol, Durgapur, Howrah etc. Its basin is characterized by the rocks consisting of granites and granitic
gneisses of Archaeans, sandstones and shales of the Gondwanas and the recent alluvial. Damodar basin is
known for its coal deposits, accounting for 46% of the country’s coal reserves. Several other minerals
23 www.visionias.in ©Vision IAS
are also associated with the geological formations of the river basin such as mica, fireclay, bauxite,
limestone, china clay, baryte, ironstone. Hence, pair 1 is correctly matched.
• McArthur River is a river in Northern Australia which flows into the Gulf of Carpentaria. It has
significant mineral reserves such as zinc, lead and silver deposits. Major ore reserves lie directly
beneath the river. The McArthur River zinc mine is based on one of the world's largest zinc, lead and
silver deposits. It was initially an underground mining operation that has been converted to open-
cut. Hence, pair 2 is correctly matched.
• Birim River is one of the main tributaries of the Pra River in Ghana. It is one of the important
production areas for gold and diamond. It is named after the Birimian rock formation, which yields
most of the gold in the region. Ghana is the second-largest producer of gold in Africa. It was also the basis
for the former name of Ghana: the Gold Coast. Hence, pair 3 is correctly matched.
Q 60.D
• Particulates pollutants are the minute solid particles or liquid droplets in the air. These are present in
vehicle emissions, smoke particles from fires, dust particles and ash from industries. Particulates in the
atmosphere may be viable or non-viable.
• The viable particulates e.g., bacteria, fungi, moulds, algae etc., are minute living organisms that
are dispersed in the atmosphere. Human beings are allergic to some of the fungi found in air. They
can also cause plant diseases. Hence the correct answer is option (d)
• Non-viable particulates may be classified according to their nature and size as follows:
o Smoke particulates consist of solid or mixture of solid and liquid particles formed during the
combustion of organic matter.
o Dust is composed of fine solid particles (over 1μm in diameter), produced during the crushing,
grinding and attribution of solid materials.
o Mists are produced by particles of spray liquids and by condensation of vapours in air. Examples are
sulphuric acid mist.
o Fumes are generally obtained by the condensation of vapours during sublimation, distillation, boiling
and several other chemical reactions.

Q 61.B
• ‘The World Migration Report 2020’ was recently published by the International Organization for
Migration (IOM).
o IOM is headquartered in Geneva, Switzerland. It was established in 1951 and is a related organization
to the United Nations.
• Highlights from the report:
o A total of approx 3.5 percent of the world population has migrated to date, making a 0.1 percent
increase from the last report published in 2017.
o The Total Number of Migrants in the year 2019 is estimated to be 270 million.
o 52 percent of international migrants were male; 48 percent were female.
o The United States remained as the top destination of the migrants with 51 million out of total
estimated 270 million. Hence, statement 2 is correct.
o Two-thirds (around 164 million people) of the migrants migrated in search of a job. Hence, statement
1 is not correct.
o 141 million, almost half of the international migrants are living in North America and Europe.
o India with a strong 17.5 million diaspora across the world, continues to be the largest country of
origin of international migrants, followed by Mexico with 11.8 million and China with 10.7 million
making the top three. Hence, statement 3 is correct.

Q 62.B
• Pitcher plants are several different carnivorous plants which have modified leaves known as pitfall
traps—a prey-trapping mechanism featuring a deep cavity filled with digestive liquid. The traps of
what are considered to be "true" pitcher plants are formed by specialized leaves. The plants attract and
drown their prey with nectar.
• The families Nepenthaceae and Sarraceniaceae are the most species-rich families of pitcher plants.
• Schedule VI of the Wild Life (Protection) Act, 1972 covers only Beddome's cycad, blue vanda, kuth,
ladies slipper orchids, the pitcher plant and red vanda.
24 www.visionias.in ©Vision IAS
• The leaf and root are used as medicine. Pitcher plant is taken by mouth for digestive disorders,
particularly constipation; for urinary tract diseases and fluid retention; as a cure for smallpox; and to
prevent scar formation. A pitcher plant extract (Sarapin) is given as a shot for relieving pain in the back,
neck, and other locations in the body. Hence the correct answer is option (b)

Q 63.A
• The earliest vertebrates were jawless fish, similar to living hagfish. They lived between 500 and 600
million years ago. Not too long after hagfish first appeared, fish similar to lampreys evolved a partial
vertebral column. The first fish with a complete vertebral column evolved about 450 million years ago.
• The first amphibians evolved from a lobe-finned fish ancestor about 365 million years ago. They were
the first vertebrates to live on land, but they had to return to water to reproduce. This meant they had to
live near bodies of water.
• The first reptiles evolved from an amphibian ancestor at least 300 million years ago. They laid amniotic
eggs and had internal fertilization. They were the first vertebrates that no longer had to return to water to
reproduce. They could live just about anywhere.
• Mammals and birds both evolved from reptile-like ancestors. The first mammals appeared about 200
million years ago and the earliest birds about 150 million years ago.
• The correct chronological order is Fishes-Amphibians- Reptiles. Hence the correct answer is option (a).

Q 64.C
• There are many different types of rays present in sunlight. The rays that are most damaging to our skin are
called ultraviolet (UV) rays. There are two basic types of ultraviolet rays that reach the earth’s surface—
UVB and UVA. UVB rays are responsible for producing sunburn.
• UVB causes non-melanoma skin cancer and plays a major role in malignant melanoma development. In
addition, UVB has been linked to the development of cataracts, a clouding of the eye’s lens.
• Indirect changes caused by UVB (such as changes in plant form, how nutrients are distributed within the
plant, timing of developmental phases and secondary metabolism) may be equally or sometimes more
important than damaging effects of UVB. These changes can have important implications for plant
competitive balance, herbivory, plant diseases, and biogeochemical cycles.
• The most severe effects are decreased reproductive capacity of animals and impaired larval
development in living organisms. Small increases in UVB exposure could result in population
reductions for small marine organisms with implications for the whole marine food chain.

Q 65.C
• World climates are generally divided into five large regions: tropical, dry, mid-latitude, high latitude, and
highland. The regions are divided into smaller subregions, some of which are described below.
• Warm Temperate Eastern Margin Climate
o Different variants of Warm Temperate Eastern Margin Climate include the Temperate monsoon
Climate or China Type Climate, Gulf Type Climate and Natal Type Climate.
o It is found between 20° and 35° N and S latitude (warm temperate latitudes just outside the tropics);
on the east coast in both hemispheres.
25 www.visionias.in ©Vision IAS
o It is characterized by a warm moist summer and a cool, dry winter with one exception: i.e in Natal
Type where winters are also moist.
o Rainfall is more than moderate, anything from 60 cm to 150 cm which is adequate for all agricultural
purposes and hence supports a wide range of crops.
o These regions support luxuriant vegetation. The lowlands carry both evergreen broad-leaved forests
and deciduous tree
o Areas that experience this climate are very densely populated. Hence, option (c) is the correct
answer.
• Tropical Monsoon Climate
o Tropical humid climates occur within 5° to 30° N and S of the equator.
o The overhead sun throughout the year and the presence of the InterTropical Convergence Zone
(ITCZ) make the climate hot and humid.
o There are three seasons: cool, dry season (October to February), hot dry season (March to mid-June),
rainy season (mid-June to September)
o The forests are more open and less luxuriant than the equatorial jungle and there are far fewer species.
o Monsoon climatic regions support high population density.
• Savanna Climate or Tropical Wet and Dry Climate or Sudan Climate
o It has alternate wet and dry seasons similar to monsoon climate but has considerably less annual
rainfall.
o There are two seasons – winter and summer.
o Days are hot and nights are cold. This extreme diurnal range of temperature is another characteristic
feature of the Sudan type of climate.
o It is typified by tall grass and short trees. The grasslands are also called as ‘bush-veld’.
o Many tribes live in savanna region. Tribes like the Masai tribes of the East African plateau are
pastoralists whereas Hausa of northern Nigeria are settled cultivators.
• Mediterranean Climate or Warm Temperate Western Margin Climate or Warm Temperate West
Coast Climate
o It is confined to the western portion of continental masses, between 30° and 45° north and south of the
equator.
o The basic cause of this type of climate is the shifting of the wind belts.
o It experiences hot, dry summers and cool, wet winters.
o Trees with small broad leaves occur with wide spacing and they are never very tall. The absence of
shade is a distinct feature of Mediterranean lands.
o The Mediterranean lands are also known as the world’s orchard lands. A wide range of citrus fruits
such as oranges, lemons, limes, citrons and grapefruit are grown.
o European Mediterranean has many ancient cities and are famous for their health and pleasure resorts,
frequented by millions all-round the year.

26 www.visionias.in ©Vision IAS


Q 66.B
• Vishnuprayag is one of the Panch Prayag (five confluences) of Alaknanda River, and lies at the
confluence of Alaknanda River and Dhauliganga River, in Chamoli district in the Indian state of
Uttarakhand. Hence pair 1 is correctly matched.
• Nandaprayag is a town and a nagar panchayat in Chamoli district in the Indian state of Uttarakhand.
Nandaprayag is one of the Panch Prayag of Alaknanda River and lies at the confluence of the Alaknanda
River and Nandakini River. Hence pair 2 is correctly matched.
• Karnaprayag is a city and municipal board in the Chamoli District in the Indian state of Uttarakhand.
Karnaprayag is one of the Panch Prayag of Alaknanda River, situated at the confluence of the Alaknanda
and Pindar River. Hence pair 3 is not correctly matched.
• Rudraprayag is a town and a municipality in Rudraprayag district in the Indian state of Uttarakhand.
Rudraprayag is one of the Panch Prayag of Alaknanda River, the point of confluence of rivers Alaknanda
and Mandakini. Kedarnath, a Hindu holy town is located 86 km from Rudraprayag. Hence pair 4 is not
correctly matched.
• Devprayag is a town and a nagar panchayat in Tehri Garhwal district in the state of Uttarakhand, India,
and is one of the Panch Prayag of Alaknanda River where Alaknanda, Saraswati River and Bhagirathi
rivers meet and take the name Ganga. Hence pair 5 is correctly matched. Hence option (b) is the
correct answer.

Q 67.A
• Palkonda Hills, series of ranges in southern Andhra Pradesh state of southern India. The hills trend
northwest to southeast and form the central part of the Eastern Ghats. Geologically, they are relics of
ancient mountains formed during the Cambrian Period (about 540 to 490 million years ago) that were
subsequently eroded by the Penneru River and its tributaries. Hence pair 1 is correctly matched.
• Kaimur Hills, also called Kaimur Range, the eastern portion of the Vindhya Range, starts near Katangi in
the Jabalpur district of Madhya Pradesh and run generally east for a distance of about 480 km to Sasaram
in Bihar. Its maximum width is about 80 km. Hence pair 2 is not correctly matched.
• Maikala Range, a mountain range in Chhattisgarh state, central India. It runs in a north-south direction
and forms the eastern base of the triangular Satpura Range. The Maikala Range consists of laterite-
capped, flat-topped plateaus (pats) with elevations ranging from 600 to 900 meters. Hence pair 3 is not
correctly matched.

Q 68.C
• Madhya Pradesh's Panna National Park has been declared a UNESCO Biosphere Reserve.
• Panna National Park is situated in the Vindhyas mountain range in the northern part of Madhya
Pradesh.
• Ken river (a tributary of Yamuna river) flows through the reserve.
27 www.visionias.in ©Vision IAS
• The region is also famous for Panna diamond mining.
• The UNESCO's recognition cited Panna Tiger Reserve as a critical tiger habitat. (As envisaged in
the Forest Rights Act, 2006, Critical tiger/Wildlife Habitats are to be declared by the Central
Government).
• The Panna Biosphere Reserve is the third in Madhya Pradesh to be included in the list after Pachmarhi
and Amarkantak. Madhya Pradesh has the highest number of tigers in the country followed by
Karnataka and Uttarakhand.
• By 2009 the entire tiger population had been eliminated by poaching subsequently tiger reintroduction
project was started here. In 2009 tigers were relocated here from Bandhavgarh National Park and
Kanha National Park.
• Hence option (c) is the correct answer.

Q 69.B
• India is one of the first countries in the world to develop a comprehensive Cooling Action plan which has
a long term vision to address the cooling requirement across sectors and lists out actions that can help
reduce the cooling demand. The cooling requirement is cross-sectoral and an essential part of
economic growth and is required across different sectors of the economy such as residential and
commercial buildings, cold-chain, refrigeration, transport, and industries
• The India Cooling Action seeks to
i. reduce cooling demand across sectors by 20% to 25% by 2037-38,
ii. reduce refrigerant demand by 25% to 30% by 2037-38,
iii. Reduce cooling energy requirements by 25% to 40% by 2037-38,
iv. recognize “cooling and related areas” as a thrust area of research under the national S&T Programme,
v. training and certification of 100,000 servicing sector technicians by 2022-23, synergizing with Skill
India Mission. These actions will have significant climate benefits.
• The Cooling Action Plan is not a part of India's Intended Nationally Determined Contribution.
Hence option (b) is the correct answer.

Q 70.B
• India has a trade surplus with the United States of America and the United Arab Emirates. Hence option
(b) is the correct answer.

Q 71.A
• Recent Context: Pakistan has been re-elected to the United Nations Human Rights Council despite
opposition from activist groups over its abysmal human rights record. Hence statement 3 is not correct.
• UN Human Rights Council (Council or HRC) was established in 2006 by UN General Assembly (UNGA)
resolution 60/251. Hence, statement 1 is correct.
• Human Rights Council replaced the former United Nations Commission on Human
Rights. The headquarters of UNHRC is in Geneva, Switzerland.
28 www.visionias.in ©Vision IAS
• It is the principle intergovernmental body within the United Nations (UN) system responsible for
strengthening the promotion and protection of human rights around the globe, and for addressing and
taking action on human rights violations around the globe.
• The HR Council is made up of 47 member States who are elected by the UN General Assembly by a
simple majority vote, through a secret ballot.
o Members of the Council are elected for three-year terms with one-third of the members being
renewed each year. Members not eligible for immediate re-election after serving two
consecutive terms i.e. they can serve 2 consecutive terms at most in a single stretch.
o Its membership is based on equitable geographical distribution of seats according to the
following regional breakdown: 13 African States; 13 Asia-Pacific States; 8 Latin American and
Caribbean States; 7 Western European and other States; 6 Eastern European States.
o All U.N. members are eligible to run for a seat on the Council.
o It holds three regular sessions per year of a total of at least ten weeks and can call special sessions to
react quickly to urgent country and thematic situations.
o Its decisions, resolutions, and recommendations are not legally binding. Hence, statement 2 is
correct.
o As a subsidiary of the General Assembly, it reports directly to the Assembly’s 193 members.
o It receives substantive and technical support from the U.N. Office of the High Commissioner for
Human Rights (OHCHR), an office within the U.N. Secretariat.
Q 72.C
• The ocean currents known since antiquity are called surface currents. Though these are invaluable to
shipping, they are superficial and occupy only a small fraction of the ocean's waters. The majority of the
ocean's currents take the form of a temperature- and salinity-driven "conveyor belt" that slowly churns
water within the abyssal depths. These loops of water circulation are called deep currents.
• Unlike the wind-driven surface currents, deep water currents are driven by differences in water density:
heavier water sinks while lighter water rises. Hence, statement 1 is correct.
• The main determinants of water density are temperature and salt concentration; thus, the deep currents are
thermohaline (temperature- and salt-driven) currents. Water at the polar latitudes sinks because it is cold
and displaces the water beneath it, pushing it along the contours of the ocean basin. Eventually, this water
pushes back up to the surface in a process called upwelling.
• Surface currents may reach several kilometers per hour and have a noticeable effect on oceanic travel.
Deep currents are much slower and may take many years to traverse the world's oceans. This movement
can be gauged by the composition of chemicals dissolved in the seawater. Hence, statement 2 is correct.
• Chemical estimates largely agree with deep current measurements and indicate that currents take up to a
thousand years to reach the surface, as seems to be the case with the North Pacific current

Q 73.D
• Volatile organic compounds (VOC) are organic chemicals that have a high vapour pressure at ordinary
room temperature. Their high vapour pressure results from a low boiling point, which causes large
numbers of molecules to evaporate or sublimate from the liquid or solid form of the compound and enter
the surrounding air, a trait known as volatility. For example, formaldehyde, which evaporates from paint
and releases from materials like resin, has a boiling point of only –19 °C (–2 °F).
• Small amounts of VOCs are produced by animals and microbes.
• The exhaled human breath contains a few thousand volatile organic compounds and is used in breath
biopsy to serve as a VOC biomarker to test for diseases such as lung cancer.
• Hence option (d) is the correct answer.

Q 74.C
• The chief gateways of the world of international trade are the harbors and ports. Cargoes and travelers
pass from one part of the world to another through these ports. The ports provide facilities of docking,
loading, unloading and storage facilities for cargo.
• These facilities are provided via arrangements of navigable channels, tugs and barges, and adequate labor
and managerial services. The importance of a port is judged by the size of cargo and the number of ships
handled. The quantity of cargo handled by a port is an indicator of the level of development of its
hinterland.
• Ports can be classified differently for example, according to types of traffic handles, on the basis of
location, on the basis of specialized functions etc.

29 www.visionias.in ©Vision IAS


• Inland Ports: These ports are located away from the sea coast. They are linked to the sea through a river
or a canal. Such ports are accessible to flat bottom ships or barges. For example, Manchester is linked
with a canal; Memphis is located on the river Mississippi; Rhine has several ports like Mannheim and
Duisburg; and Kolkata is located on the river Hoogli, a branch of the river Ganga. Inland ports are “wet
ports.” located on a river or lake and not to be confused with dry ports. Hence, pair 2 is not correctly
matched.
• Dry Port: A dry port is an intermodal terminal directly connected to a seaport by road or rail. It operates
as a centre for the transshipment of sea cargo to inland destinations. Dry ports are not located on a body of
water. In fact, they can be hundreds of kilometers from the nearest ocean or river. Some of the dry ports
are Viramgam in Gujarat, Durgapur in West Bengal, Faisalabad and Lahore in Pakistan, Riyadh in Saudi
Arabia etc. Hence, pair 1 is not correctly matched.
• Warm-water port: It is one where the water does not freeze in wintertime. Since they are available year-
round, warm-water ports can be of great geopolitical or economic interest in regions having severe
winters. Settlements such as Dalian in China, Vostochny Port, Murmansk and Petropavlovsk-Kamchatsky
in Russia, Odessa in Ukraine, Kushiro in Japan and Valdez at the terminus of the Alaska Pipeline owe
their very existence to being ice-free ports. Hence, pair 3 is correctly matched.
• Out Ports: These are deep water ports built away from the actual ports. These serve the parent ports by
receiving those ships which are unable to approach them due to their large size. A classic combination, for
example, is Athens and its outport Piraeus in Greece.

Q 75.D
• Major threats to freshwater fishes and other freshwater biodiversity include habitat modification,
fragmentation, and destruction; invasive species; overfishing; environmental pollution; forestry practice;
and climate change.
o Habitat modification, fragmentation and destruction: The amount of water impounded behind
dams has quadrupled since 1960, and three to six times more water is held in reservoirs than in natural
rivers. Water withdrawals from rivers and lakes have doubled since 1960, and 70% of all water used
worldwide is for agriculture. This has taken a heavy toll on the world"s freshwater biota, in general,
causing many species of freshwater fish to become endangered.
▪ Large dams fragment rivers and habitats, isolating species, interrupting the exchange of
nutrients between ecosystems, and cutting off migration routes. They reduce water and
sediment flows to downstream habitat and can decimate a river's estuary, where many of the
world's fish species spawn. Dams also increase ecosystems’ vulnerability to threats like climate
change.
o Invasive species: After habitat destruction, the introduction of exotic or "alien" invasive species is
believed to be one of the greatest global threats to native fish communities and their freshwater
ecosystems. Many introductions are intentional to provide new fisheries (Nile perch in Lake Victoria),
for the commercial or cultural purpose (tilapia), as ornamental species (koi carp) or to control
mosquito larvae as a biomedical measure against malaria (mosquito fish). Whatever the source, many
of these aliens have caused problems worldwide, including the destruction of habitat, introduction of
disease and parasites, and predation on or competition with native species of fish.
o Overfishing: Overexploitation (unsustainable fishing) poses a serious threat to fish and aquatic
biodiversity and also to the livelihoods of people in riverine and lake communities. Major causes are
excessive food demands, market pressures, fishing gear technology development, weak or lack of
appropriate management approaches and policies, accidental by-catches, and an unregulated aquarium
trade in wild species.
o Environmental pollution: The outcome of pollution on freshwaters, whether as a result of industrial,
domestic or agricultural activities, is often catastrophic and can result in the elimination of fish
species, and dead rivers and lakes.In the face of environmental pollution, there is one source which
does not immediately spring to people"s minds, but which does have a major impact on freshwater
fish populations: the pharmaceutical products, such as antidepressants and birth-control pills that end
up in human waste and eventually in freshwaters.
o Forestry practice: The impact of forestry practice – both afforestation and deforestation – on
freshwater habitats and fishes is a cause of much concern. Deforested streams also reduce woody
debris in-channel input, thus simplifying channel morphological and hydrological complexity. The
lack of debris reduces rearing and food habitat availability. There can be reduced water temperatures
in afforested streams where the channel is shaded and increased water temperatures where the
catchment has been deforested. The leaves dropping from forest canopies will affect stream
chemistry. Such physical and chemical effects combine in various ways to affect the plants,
30 www.visionias.in ©Vision IAS
invertebrates and freshwater fishes in areas impacted by forestry. Changes in hydrology, ambient
water temperatures and extreme fluctuations in chemistry tend to make conditions in streams too
extreme for most biota.
o Hence, option (d) is correct.

Q 76.C
The National Action Plan on Climate change (NAPCC) was formally launched on June 30th, 2008. The
NAPCC identifies measures that promote development objectives while also yielding co-benefits for
addressing climate change effectively. There are eight “National Missions” which form the core of the
National action plan. They focus on promoting understanding of climate change, adaptation and
mitigation, energy efficiency, and natural resource conservation.”
The eight missions are:
1. National Solar Mission
2. National Mission for Enhanced Energy Efficiency (NMEEE )
3. National Mission on Sustainable Habitat
4. National Water Mission
5. National Mission for Sustaining the Himalayan Ecosystem
6. National Mission for a Green India
7. National Mission for Sustainable Agriculture
8. National Mission on Strategic Knowledge for Climate Change
NMEEE consist of four initiatives to enhance energy efficiency in energy-intensive industries which are
as follows:
1. Perform Achieve and Trade Scheme (PAT) - Implementing a market assisted compliance
mechanism to accelerate the implementation of cost-effective improvements in energy efficiency in
large energy-intensive industries.
2. Market Transformation for Energy Efficiency (MTEE) - Accelerating the shift to energy-efficient
appliances in a specific application through innovative measures to make the products more
affordable.
3. Energy Efficiency Financing Platform (EEFP) - Facilitating Financial Institutions to invest in
Energy Efficiency Projects and Programmes
4. Framework for Energy Efficient Economic Development (FEEED) - Developing fiscal
instruments to leverage financing for Energy Efficiency through risk mitigation:
1. Partial Risk Guarantee Fund for Energy Efficiency (PRGFEE) and
2. Venture Capital Fund for Energy Efficiency (VCFEE) to promote energy efficiency
3. National Energy Conservation Award and Painting Competition

Q 77.D
• In FY 2018-19, the total gas consumption in India was around 148.02 MMSCMD. The share of domestic
gas and imported RLNG was about 48% & 52% respectively. Hence, statement 2 is not correct.
• Sector-wise gas consumption in FY 2018-19 is tabulated as under:

31 www.visionias.in ©Vision IAS


Q 84.C
• An initiative was launched by WHO to eliminate Visceral leishmaniasis as a public health problem from
the South East Asia region by 2020. The deadline has now been extended to 2023.
• Kala Azar, also known as Visceral leishmaniasis, Black fever, and Dumdum fever, is the most severe
form of leishmaniasis and, without proper diagnosis and treatment, is associated with high fatality.
• Leishmaniasis is a disease caused by protozoan parasites of the Leishmania genus.
• It is a vector-borne disease and transmitted through sand flies. The parasite migrates to the internal
organs such as the liver, spleen (hence 'visceral'), and bone marrow, and, if left untreated, will almost
always result in the death of the host.
• Signs and symptoms include fever, weight loss, fatigue, anemia, and substantial swelling of the liver and
spleen.
• It is endemic to the Indian subcontinent spanning over 119 districts in the countries of Nepal, Bhutan,
Bangladesh and India. India accounts for half the global burden of the disease. It is endemic to a
contiguous block of districts spanning West Bengal, Bihar, Jharkhand and eastern Uttar Pradesh.
• It is detected by K39 Test. The K39 dipstick test is easy to perform, and village health workers can be
easily trained to use it. The kit may be stored at ambient temperature and no additional equipment needs
to be carried to remote areas.
• Related Information: There are 3 main forms of leishmaniases – visceral (also known as kala-azar,
which is and the most serious form of the disease), cutaneous (the most common), and mucocutaneous.
o Cutaneous leishmaniasis (CL) is the most common form of leishmaniasis and causes skin lesions,
mainly ulcers, on exposed parts of the body, leaving life-long scars and serious disability or stigma.
About 95% of CL cases occur in the Americas, the Mediterranean basin, the Middle East and Central
Asia. In 2018 over 85% of new CL cases occurred in 10 countries: Afghanistan, Algeria, Bolivia,
Brazil, Colombia, Iran (Islamic Republic of), Iraq, Pakistan, the Syrian Arab Republic and Tunisia. It
is estimated that between 600 000 to 1 million new cases occur worldwide annually.
o Mucocutaneous leishmaniasis leads to partial or total destruction of mucous membranes of the nose,
mouth and throat. Over 90% of mucocutaneous leishmaniasis cases occur in Bolivia (the Plurinational
State of), Brazil, Ethiopia and Peru.

Q 85.A
• The Climate Vulnerable Forum (CVF) is an international partnership of countries highly vulnerable to a
warming planet. Hence statement 1 is correct.
• The Forum serves as a South-South cooperation platform for participating governments to act together
to deal with global climate change. Hence statement 2 is not correct.
• CVF is the international forum for countries most threatened by climate change. Composed of 48
members from Africa, Asia, the Caribbean, Latin America, and the Pacific, it represents some 1.2 billion
people worldwide. It was founded in November 2009 by the Maldives at Male’, together with 10 other
countries. The Forum is led by a rotating chair for an ordinary period of two years, with Bangladesh
currently chairing for the second time for the period 2020-2022.

Q 86.A
• The Kashmir stag, also called hangul, is a subspecies of elk native to Kashmir. It is found in dense
riverine forests in the high valleys and mountains of the Kashmir Valley and northern Chamba
district in Himachal Pradesh.
• Dachigam National Park has located 22 km (kilometer) from Srinagar in Jammu and Kashmir. The
park is best known as the home of the hangul, or Kashmir stag. It covers an area of 141 sq km (square
kilometers). The name literally stands for ‘ten villages’, which could be in memory of the ten villages that
were relocated in order to create the park.
• Dachigam was initially established to ensure the supply of clean drinking water to Srinagar city. A
protected area since 1910, it was declared as a national park in 1981.
• These deer once numbered about 5,000 animals at the beginning of the 20th century. Unfortunately, they
were threatened, due to habitat destruction, over-grazing by domestic livestock, and poaching. This
dwindled to as low as 150 animals by 1970. However, the state of Jammu & Kashmir, along with the
IUCN and the WWF prepared a project for the protection of these animals. It became known as Project
Hangul. This brought great results and the population increased to over 340 by 1980. It is listed as
critically endangered on the IUCN Red List. Hence the correct answer is option (a)
• Tal Chhapar Sanctuary is a sanctuary located in the Churu district of Northwestern Rajasthan in the
Shekhawati region of India. It is known for blackbucks and is also home to a variety of birds.
34 www.visionias.in ©Vision IAS
• The Mudumalai National Park and Wildlife Sanctuary also a declared tiger reserve, lies on the
northwestern side of the Nilgiri Hills (Blue Mountains), in Nilgiri District. The sanctuary is divided into
five ranges – Masinagudi, Thepakadu, Mudumalai, Kargudi and Nellakota.
The protected area is home to several endangered and vulnerable species including the Indian elephant,
Bengal tiger, gaur and Indian leopard. There are at least 266 species of birds in the sanctuary, including
critically endangered Indian white-rumped vulture and long-billed vulture
• Kanha Tiger Reserve, also known as Kanha–Kisli National Park, is one of the tiger reserves of India
and the largest national park of the state of Madhya Pradesh.

Q 87.B
• Statement 1 is not correct: Uttar Pradesh (199.5 million) is the most populous state in the country
followed by Maharashtra with 112 million.
• Statement 2 is correct: Kerala with 1084 has the highest sex ratio followed by Puducherry with 1038,
Damand and Diu has the lowest sex ratio of 618.
• Statement 3 is correct: Percentage growth n literacy during 2001-2011 is 38.82% with males being
around 31.98% and females being 49.10%.
• Some other highlights of census 2011 are:
o Total population of India has reached to 121 crores. It comprises 62.31 crores males and 58.74 crores
females.
o Total absolute increase in population during the decade is 18.19 crores.
o Population growth during the decade remained 17.7 percent.
o Density of population per sq. km. has been 382 persons.
o Sex ratio for females per 1000 males is 943 females. Child sex ratio for females is 919 per 1000
males.
o Literacy rate of India has gone up to 73.0 percent from previous figure of 64.83 percent.

Q 88.D
• Ash produced by thermal power plants is a proven resource material for many applications of
construction industries and currently is being utilized in Manufacture of Portland Pozzolana
Cement (PPC), fly ash bricks/blocks/tiles manufacturing, road embankment construction & low
lying area development, in agriculture as soil conditioner etc. Hence the correct answer is option (d).
• Further, following efforts have been made to make optimum utilization of fly ash as an environmentally
sustainable and economically viable product:
o GST rates on fly ash and its products have been reduced to 5%.
o To facilitate 100% ash utilization by all coal based thermal power plants, a web portal for
monitoring of fly ash generation and utilization data of Thermal Power Plants and a mobile based
application titled “ASHTRACK” has been launched by the Government that will help to establish a
link between fly ash users and power plants executives for obtaining fly ash for its use in various
areas.
o A Workshop and Video Conferences to make use of the Web Page for data updation have been
conducted by CEA and NTPC.
o Ash-park has been developed and awareness programme for utilisation of fly ash and its products
have been conducted.

Q 89.C
• Nano fertilizers are biologically synthesized from waste and natural mineral materials using indigenous
microbial resources. As they are made from natural materials, they do not have the toxic side effects of
chemical fertilizers.
• The nano nutrients are required in quantities of just a few grams per acre as compared to chemical
fertilizers that are required in kilograms per acre.
• No dangers of fertilizer runoffs that contaminate natural water bodies and adversely impact human health
and environment and the scale of efficacy from use in minuscule amounts make it a disruptive technology.
• The TERI-Deakin Nanobiotechnology Centre (TDNBC) has produced these fertilizers at its center in
Gurugram. These products contain efficiently stabilized essential nutrients in a nanostructured form (size
range of 50-100 nm) that provides higher use efficiency and slow release in nature.
• Hence option (c) is the correct answer.

35 www.visionias.in ©Vision IAS


Q 90.D
• Recently, The Royal Society unveiled a new portrait of astrophysicist Dame Jocelyn Bell Burnell,
who is credited with discovering pulsars in 1967. Discovery was recognised by a Nobel Prize in physics
in 1974 that was shared by two professors, Antony Hewish (Burnell’s supervisor) and Martin Ryle.
• Pulsars belong to a family of objects called neutron stars that form when a star more massive than the sun
runs out of fuel in its core and collapses in on itself. This stellar death typically creates a massive
explosion called a supernova.
o The neutron star is the dense nugget of material left over after this explosive death. The only object
with a higher density than a neutron star is a black hole, which also forms when a dying star collapse.
• The pulsars are rapidly rotating neutron stars that emit radio-frequency pulses. Hence statement 1
is correct.
o Pulsars spin because the stars from which they formed also rotate, and the collapse of the stellar
material will naturally increase the pulsar's rotation speed.
• Pulsars are highly magnetic. Pulsars have magnetic fields that range from 100 million times to 1
quadrillion (a million billion) times stronger than Earth's. Hence statement 2 is correct.
• Pulsars can radiate light in multiple wavelengths, from radio waves all the way up to gamma-rays, the
most energetic form of light in the universe.
o Also, the beam of radio waves emitted by a pulsar may not pass through the field of view of an Earth-
based telescope, preventing astronomers from seeing it.
• Scientists use pulsars to study extreme states of matter, gravitational waves, search for planets
beyond Earth's solar system and measure cosmic distances. Hence statement 3 is correct.
• Additional Information: The International Pulsar Timing Array (IPTA) is a collaboration of radio
astronomers from a dozen countries across the globe. It uses more than 12 radio telescopes all over the
world with an aim to detect low frequency gravitational waves.
o The goal of the IPTA is to detect and characterize the low-frequency gravitational wave
universe through timing a global array of approximately 100-millisecond pulsars using the
largest radio telescopes in the world. Through sharing resources and creating combined pulsar
timing data sets, the IPTA is constructing the most sensitive low-frequency gravitational wave
detector possible. India has been an associate member of the IPTA since the last four years.
o IPTA comprises of the European Pulsar Timing Array (EPTA), the North American Nanohertz
Observatory for Gravitational Waves (NANOGrav), and the Parkes Pulsar Timing Array (PPTA).
o India is its associate member.

Q 91.C
• Mixed farming is one of the agriculture methods in which more than one agricultural practices
takes place on the same agricultural field
• Traditional farmers can often be characterised as managing mixed subsistence economies; employing
several cropping systems simultaneously. This includes having plots under continuous cultivation,
some in shifting cultivation and also raising animals. This allows for a larger diversity of outputs,
greater flexibility and increased system stability. Hence statement 1 is correct.
• Advantages of Mixed Farming:
o In this type of farming field will not be bare at all. They would be producing some or the other crop
continuously.
o Improve productivity of the land in the farm.
o One type of farming can support the other, which will reduce the cost of food and manure.
o Higher Profits
o Animal husbandry povides manure and the livestock also helps in clearing agricultural waste
o Even if one crop fails, the farmer can compensate the loss with the other crop. Thus provides farmers
with a stable source of income. Hence statement 3 is correct.
• Disadvantages of Mixed Farming:
o This type of farming is tougher to maintain than the monoculture type. This is due to large number of
activities involved in mixed farming. Monitoring and maintaining of mixed farming should be very
careful and hence it is difficult.
o A disease in one crop may even affect the other crop.

36 www.visionias.in ©Vision IAS


o The farmers should have complete knowledge of mixed farming. As there are number of activities
involved in mixed farming it is better if the farmer has good knowledge of different sections of
farming. Hence statement 2 is not correct.
o The main drawback of mixed farming is its capacity to grow crops. Growing multiple crops on
already limited land is a very big challenge.
Q 92.C
• During an El Niño event, the surface waters in the central and eastern Pacific Ocean become significantly
warmer than usual. That change is intimately tied to the atmosphere and to the winds blowing over the
vast Pacific. Easterly trade winds (which blow from the Americas toward Asia) falter and can even
turn around into westerlies. This allows great masses of warm water to slosh from the western Pacific
toward the Americas. Hence, option 2 is not correct.
• It also reduces the upwelling of cooler, nutrient-rich waters from the deep—shutting down or reversing
ocean currents along the equator and along the west coast of South and Central America. Hence,
option 3 is correct.
• The strengthening and weakening of the trade winds is a function of changes in the pressure gradient of
the atmosphere over the tropical Pacific.
• In general, warm El Niño events are characterized by more tropical storms and hurricanes in the
eastern Pacific and a decrease in the Atlantic, Gulf of Mexico and the Caribbean Sea. Hence, option 1 is
correct.
o The primary explanation for the decline in hurricane frequency during El Niño years is due to the
increased wind shear in the environment. In El Niño years, the wind patterns are aligned in such a
way that the vertical wind shear is increased over the Caribbean and Atlantic. The increased wind
shear helps to prevent tropical disturbances from developing into hurricanes. In the eastern Pacific,
the wind patterns are altered in such a way to reduce the wind shear in the atmosphere, contributing to
more storms.
Q 93.B
• Seafloor spreading is a process that occurs at mid-ocean ridges, where new oceanic crust is formed
through volcanic activity and then gradually moves away from the ridge.
• Seafloor spreading and other tectonic activity processes are the results of mantle convection. Mantle
convection is the slow, churning motion of Earth’s mantle. Convection currents carry heat from the lower
mantle and core to the lithosphere. Convection currents also “recycle” lithospheric materials back to the
mantle. Hence, statement 1 is not correct.
• Seafloor spreading occurs along mid-ocean ridges—large mountain ranges rising from the ocean floor.
Seafloor spreading occurs at divergent plate boundaries. As tectonic plates slowly move away from each
other, heat from the mantle’s convection currents makes the crust more plastic and less dense. The less-
dense material rises, often forming a mountain or elevated area of the seafloor. Hence, statement 2 is
correct.
• Eventually, the crust cracks. Hot magma fueled by mantle convection bubbles up to fill these fractures and
spills onto the crust. This bubbled-up magma is cooled by frigid seawater to form igneous rock. This rock
(basalt) becomes a new part of Earth’s crust.
• Seafloor spreading is not consistent at all mid-ocean ridges. Spreading rate is the rate at which an ocean
basin widens due to seafloor spreading. Spreading rates determine if the ridge is fast, intermediate, or
slow. As a general rule, fast ridges have spreading (opening) rates of more than 90 mm/year. Intermediate
ridges have a spreading rate of 40–90 mm/year while slow-spreading ridges have a rate less than 40
mm/year.
• Slowly spreading ridges are the sites of tall, narrow underwater cliffs and mountains. Rapidly spreading
ridges have a much more gentle slope. Hence, statement 3 is correct.

Q 94.D
• 2020 Noble Peace Prize awarded to the World Food Programme (WFP).
• World Food Programme is the leading humanitarian organisation saving lives and changing lives,
delivering food assistance in emergencies and working with communities to improve nutrition and build
resilience.
• It was founded in 1961 by the Food and Agriculture Organisation (FAO) and United Nations
General Assembly (UNGA) with its headquarters in Rome, Italy. Hence statement 1 is not correct.
• The World Food Programme has no independent source of funds, it is funded entirely by voluntary
donations. Its principal donors are governments, but the organization also receives donations from the
private sector and individuals. Hence statement 2 is not correct.

37 www.visionias.in ©Vision IAS


• Global Report on Food Crisis is a report released by WFP.
• WFP is also a member of the United Nations Sustainable Development Group.
• WFP’s role in India:
o The WFP has been working in India since 1963, two years after its establishment. it provides policy
inputs, advocacy and technical assistance for improving access to food.
o The WFP has proposed some initiatives like Automatic Grain Dispensing Machine
(Annapurti) and Mobile Storage Units for the effective implementation of TPDS. Annapurti
allows beneficiaries to withdraw their foodgrain quota accurately and at a time of their choice. It can
dispense two commodities at a speed of 25 kg per 1.3 minutes. It has a storage capacity of 200 kg to
500 kg.
o It has completed a pilot on rice fortification used in the government’s Mid-day Meals scheme in
Varanasi.
o During the pandemic, WFP signed an MoU with the Uttar Pradesh State Rural Livelihood Mission.
Under the agreement, WFP will provide technical assistance for setting up supplementary nutrition
production units in 18 districts for supply of quality food to about 33 lakh beneficiaries of
the Anganwadi scheme (Integrated Child Development Services).

Q 95.D
• Wetlands are areas where water covers the soil or is present either at or near the surface of the soil
all year or for varying periods of time during the year, including during the growing season. Water
saturation (hydrology) largely determines how the soil develops and the types of plant and animal
communities living in and on the soil.
• The different types of wetlands are:
• A carr is a type of waterlogged wooded terrain that, typically, represents a succession stage between
the original reedy marsh and the likely eventual formation of forest in a sub-maritime climate. Carrs are
wetlands that are dominated by shrubs rather than trees. The carr is one stage in a hydrosere: the
progression of vegetation beginning from a terrain submerged by freshwater along a river or lake margin.
• A bog or bogland is a wetland that accumulates peat, a deposit of dead plant material—often
mosses, and in a majority of cases, sphagnum moss. Other names for bogs include mire, mosses,
quagmire, and muskeg; alkaline mires are called fens.
• A marsh is a wetland that is dominated by herbaceous rather than woody plant species. Marshes can
often be found at the edges of lakes and streams, where they form a transition between the aquatic and
terrestrial ecosystems. They are often dominated by grasses, rushes, or reeds. Hence the correct answer
is option (d).

Q 96.D
• Statement 1 is not correct: There is heavy use of fertilisers such as urea to maintain nitrogen (N) levels,
and diammonium phosphate (DAP) and muriate of potash (MOP) for phosphorous (P) and potassium (K),
respectively. This has led to a serious imbalance in the nutrient level of Indian agricultural land where the
NPK ratio is 6.7:2.4:1, instead of the accepted 4:2:1.
• Statement 2 is not correct: As of now, the country has achieved 80% self-sufficiency in the production
capacity of Urea. As a result, India could manage its substantial requirement of nitrogenous fertilizers
through the indigenous industry besides imports. Similarly, 50% indigenous capacity has been developed
in respect of phosphatic fertilizers to meet domestic requirements. However, the raw materials and
intermediates for the same are largely imported. For potash (K), since there are no viable sources/reserves
in the country, its entire requirement is met through imports.
• Statement 3 is not correct: The requirement/ demand for fertilizers for Kharif and Rabi season is
assessed in biannual Zonal Conferences held by the Department of Agriculture, Cooperation & Farmers
Welfare (DAC&FW). The assessed requirement of Urea for Kharif 2019 was 156.22 LMT (without
reserve allocation). The assessed requirement of Urea for Rabi 2019-20 (upto Jan’20) was 146.07 LMT
(vis-à-vis 120.28 LMT for January’19) and for the complete season of Rabi 2019-20 is 179.04 LMT
(without reserve allocation). As can be viewed from the table below, there is not much difference between
the urea requirement of Rabi ad Kharif season in India.

38 www.visionias.in ©Vision IAS


Q 97.C
• Shipping canals are waterways specifically built along major seawater routes to enable the passage of
vessels. Most of the time, these canals are constructed to connect to waterbodies including seas, lakes, and
rivers, offering an alternative route to the vessels, particularly cargo vessels. Such canals are of vital
importance in the maritime industry as they offer shorter transportation routes across major seawater
networks and also help to regulate maritime traffic internally within countries.
• Some of these canals are also the busiest traffic routes around the world. Some of the significant and
busiest shipping canals are described below.
• Suez Canal: It is an artificial sea-level waterway located in Egypt. It links the Mediterranean Sea with the
Gulf of Suez. It is one of the most heavily used shipping routes in the world. It provides a shortest
maritime route between Europe and the regions which share a border with the Indian Ocean and the
Western Pacific Ocean. Hence, pair 3 is not correctly matched.
• Panama Canal: It provides connectivity between the Pacific and the Atlantic Ocean through the Panama
isthmus- a narrow strip that separates the Caribbean Sea from the Pacific Ocean. It was opened in the year
1914. It helps vessels transiting between the east and west coasts of the US to shorten their journey by
15,000 km. Hence, pair 4 is correctly matched.
• Corinth Canal: It connects the Gulf of Corinth and the Saronic Gulf in the Aegean Sea. It divides the
Peloponnese peninsula from the Greek mainland. With the 6.4-kilometre length and 8 meters (26 ft)
depth, it is considered as one of the deepest canal in the world. Itt helps the seafarers to avoid the dangers
of sailing around the Peloponnese’s treacherous southern capes. In recent times, its economic importance
has reduced due to the incapability to accommodate massive modern ships. Hence, pair 1 is not correctly
matched.
• Kiel Canal: It connects the Baltic Sea with the North Sea. It passes through the German province of
Schleswig-Holstein. It helps vessels to bypass the longer route that passes via Denmark which is regarded
as quite unstable maritime route, saving an average of 250 nautical miles. With this artificial waterway,
vessels en route to east through the North Sea enter the canal at Brunsbüttel and complete the journey at
Kiel-Holtenau to enter into the Baltic. Hence, pair 2 is correctly matched.
• The White Sea-Baltic Sea Canal: The White Sea-Baltic Sea Canal, also called White Sea Canal, is an
important waterway that regularizes traffic internally along the Russian waterways starting from the
39 www.visionias.in ©Vision IAS
White Sea in the north and extending to the Baltic Sea down south. It passes through various smaller
water bodies, including the Arctic Ocean and Lake Onega, before finally emerging into the Baltic Sea.
• Beijing-Hangzhou Grand Canal: It is popularly known as Grand Canal. It is the longest and the oldest
canal in the world connecting China’s the Yellow River and Yangtze River. It links northern and southern
China, the canal contributes heavily to the country’s economy and is recognized as a UNESCO World
Heritage Site.
• Danube-Black Sea Canal: It connects the Danube River to the Black Sea. It also interlinks the Black Sea
to the North Sea through the Danube-Main-Rhine channel. It enables vessels to bypass the difficult deltaic
region of Danube.
• Rhine-Main-Danube Canal: It links three important rivers in Western Europe, the Rhine-Main-Danube.
It is major transportation gateway linking the North Sea to the Black Sea, via the Atlantic Ocean.

Q 98.B
• These are some of the facilities for Carbon capture and storage (CCS).
• Century Plant - It is owned by Occidental Petroleum, the Century natural gas processing facility in West
Texas, US, is the world’s single biggest CCS plant.
• Shute Creek Gas Processing Plant- Owned by ExxonMobil, the Shute Creek gas processing plant is
located in Wyoming, US. The CCS facility, built near LaBarge, Lincoln County, captures approximately
365 million cubic feet per day (Mcfd) of CO2, which is equivalent to removing more than 1.5 million cars
off the road.
• Great Plains Synfuels Plant – The Great Plains Synfuels Plant is owned and operated by Dakota
Gasification, was built near Beulah, North Dakota, with an investment of $2.1bn (£1.68bn). The facility
captures and compresses up to three million tonnes of CO2 a year. The CO2 captured by the facility is a
byproduct from a coal gasification process that produces natural gas from coal.
• Carbon capture and storage (CCS) facilities are considered as key contributors to emissions reduction,
with 18 of the world’s large-scale CCS plants currently capturing approximately 40 million tonnes per
annum (Mtpa) of carbon dioxide (CO2), according to The Global CCS Institute.
• Hence option (b) is the correct answer.

Q 99.B
• The Suri is a sedentary pastoral tribe in southwestern Ethiopia. They live on the west bank of the
Omo River. The Suri tribe has a cattle-centered culture. They breed their cattle, mainly cows, on their
land in the Omo Valley. The total population of the Suri is estimated at 7,500.
• The Suri have a fierce culture, with a preference for stick fighting, also known as Donga. It’s part of
an important ritual, where young men from various Suri villages duel with sticks. There are several
referees present to ensure that all rules are followed; it’s not allowed to hit anyone on the ground, for
example. At the end of the battles, it’s customary to arrange marriages between the parents of the young
boy and a girl who has chosen him as her favorite duelist.
• Piercings and lip plates are an important feature of the Suri culture. A lip plate is a sign of beauty. A
woman is worth a larger dowry when the lip plate is bigger. Most women have their lower teeth removed
during puberty, in order to get their lower lip pierced. As soon as the lip is pierced, it will be stretched,
and a lip plate will be placed in the hole. It is still unknown why the Suri have started wearing lip plates.
One theory claims that lip plates were used to discourage slavery amongst Suri women.
• Their territory is threatened by modernization. A massive hydroelectric dam, Gibe III is being built over
the Omo River and forces the indigenous tribes of the Omo Valley to move away from their land. Results
of this dam are conflicts amongst neighboring tribes over land and cattle.
• Hence, option (b) is the correct answer.

Q 100.C
• Tropical cyclones form and intensify under a specific set of necessary conditions. These conditions must
be met in order to a tropical cyclone to form; however, the presence of these conditions does not
guarantee development. Necessary conditions for its development are discussed below.
• Warm ocean waters
o Tropical cyclones thrive off the heat content of the warm tropical and subtropical oceans. Warm
ocean water (at least 26°C) evaporates from the surface and is taken in by the developing cyclone. As
the water vapor rises, it eventually condenses into liquid water, forming clouds and releasing energy
in the form of heat. Tropical cyclones use the released energy to gain organization and strength.

40 www.visionias.in ©Vision IAS


o Thus the warmer the water, the more energy can be extracted during condensation, and the stronger
the storm can grow. However, tropical cyclones require a large temperature difference between the
ocean surface and upper-level air to be most efficient at turning the heat energy into mechanical
energy (in the form of strong winds). Hence option 1 is correct.
• Moisture in the mid-troposphere
o Tropical cyclones require a moist mid-troposphere to help keep their clouds and thunderstorms intact.
Dry air introduced into the mid-levels will begin eating away at the cyclone's clouds. Furthermore, the
dry air can become wrapped into the circulation of the cyclone, disrupting the thunderstorms and
causing asymmetries in the structure of the cyclone. Hence option 4 is correct.
• Unstable Conditions
o Tropical cyclones contain thunderstorms, which ingest moist air near the surface. The most effective
way to bring the air near the surface up into the thunderstorms is if that air is unstable. Unstable air is
typically very warm, moist air (such as over the tropical oceans) that when lifted will continue to rise
on its own. The more unstable the air is, the more rapidly it will rise. Rapidly rising air helps to create
vigorous thunderstorms which contribute to the overall tropical cyclone. Hence, option 2 is not
correct.
• Pre-existing disturbance
o A pre-existing disturbance can be considered a seedling that, if placed along with other favorable
conditions, may grow to be a tropical cyclone. It can exist in the form of easterly wave disturbances in
the Inter-Tropical Convergence Zone (ITCZ). Small local differences in the temperature of the water
and of air produce various low-pressure centers of small size. A weak cyclonic circulation develops
around these areas. Then, because of the rising warm humid air, a true cyclonic vortex may develop
very rapidly. However, only a few of these disturbances develop into cyclones.
• Low vertical wind shear
o Vertical wind shear is the difference in wind speed and direction at two different heights in the
atmosphere. An example of shear would be to have wind increasing rapidly with height. Tropical
cyclones have trouble developing in high shear environments because it displaces the thunderstorms
in the cyclone away from its center of circulation. The thunderstorms are the primary location for the
cyclone to gain energy through condensation (thinking back to the first criteria). By moving the
thunderstorms away from the center of circulation, the rotation weakens because it is not receiving a
steady supply of energy.
• At least 5° latitude from the equator
o Tropical cyclones are well known for their rotation, which is easily visible from a satellite loop of the
storm. However, in order to acquire this spin, they must obtain it from the rotation of the earth. At the
equator, the influence of the earth's spin is zero and it increases moving toward the poles (this is
known as Coriolis force). While there is nothing magical about 5º degrees latitude, there is just
enough influence from the earth's spin to be favorable for tropical cyclone development.
• Thermally contrasting air masses are a prerequisite for the formation of temperate cyclones. Hence,
option 3 is not correct.

41 www.visionias.in ©Vision IAS

You might also like